114
THEME : URINE PLASMA (U/P) OSMOLARITY RATIO A High urine Na+ and high U/P ratio B High urine Na+ and low U/P ratio C Low urine Na+ and high U/P ratio D Low urine Na+ and low U/P ratio E Normal U/P + high creatinine clearance F Normal U/P + low creatinine clearance For each of the cases described below, select the single most appropriate option from those listed above. Each option may be used once, more than once, or not at all. In pre-renal failure, the urine sodium is low and the U/P osmolarity ratio is high. In renal failure due to intrinsic disease or damage, the urine sodium is high as there is a failure of absorption and thus the U/P ratio is low. Scenario 1 Acute cortical necrosis. 0 B Correct answer Scenario 2 Water depletion. 0 C Correct answer Scenario 3

301 350

Embed Size (px)

Citation preview

THEME: URINE PLASMA (U/P) OSMOLARITY RATIO

A High urine Na+ and high U/P ratioB High urine Na+ and low U/P ratioC Low urine Na+ and high U/P ratioD Low urine Na+ and low U/P ratioE Normal U/P + high creatinine clearanceF Normal U/P + low creatinine clearance

For each of the cases described below, select the single most appropriate option from those listed above. Each option may be used once, more than once, or not at all.

In pre-renal failure, the urine sodium is low and the U/P osmolarity ratio is high. In renal failure due to intrinsic disease or damage, the urine sodium is high as there is a failure of absorption and thus the U/P ratio is low.

Scenario 1

Acute cortical necrosis.

0

B Correct answer

Scenario 2

Water depletion.

0

C Correct answer

Scenario 3

Partial ureteric obstruction.

0

F Correct answer

Scenario 4

Hypovolaemia due to bleeding.

0

C Correct answer

THEME: Lower limb ischaemia

A Femoropopliteal bypassB Percutaneous balloon angioplastyC Femorodistal bypassD Below knee amputationE Tissue plasminogen activator infusion (intra-arterial)F FasciotomyG No treatment but recommend lifestyle changes

For each of the presentations below, select the most likely single treatment from the options listed above. Each option may be used once, more than once or not at all.

 

Scenario 1

A 73-year-old diabetic woman presents with critical ischaemia of the right lower leg. Angiography reveals extensive disease of the superficial femoral, popliteal and tibial arteries. Pulse-generated run-off assessment indicates a good run-off in the posterior tibial artery.

0

C Correct answer

Femorodistal bypass

Femorodistal bypass using an autogenous vein graft is the treatment of choice in this patient. The absence of images of the posterior tibial, anterior tibial and common peroneal arteries on angiography films must not be accepted as evidence of their occlusion.

Scenario 2

A 72-year-old man presents with a 4-h history of acute ischaemia of the left leg. Clinical examinations demonstrate signs of acute ischaemia with no evidence of gangrene. There is no neurological deficit. An urgent angiogram reveals a complete occlusion of the distal superficial femoral artery probably due to thrombosis.

0

E Correct answer

Tissue plasminogen activator infusion (intra-arterial)

Intra-arterial thrombolysis with tissue plasminogen activator (TPA) or streptokinase is the treatment of choice in this patient in view of the short history (4 h), absence of neurological deficit and angiographic findings.

Scenario 3

A 57-year-old smoker presents with intermittent claudication of the right calf. The claudication distance is 70 m. Angiography reveals a 12 cm stenosis in the proximal superficial femoral artery.

0

A Correct answer

Femoropopliteal bypass

Such a long stenosis is unsuitable for angioplasty. Femorodistal bypass using an autogenous vein or synthetic graft is the treatment of choice.

Scenario 4

A 21-year-old motorcyclist presents with multiple injuries following a road traffic accident. Clinical examination reveals a critically ischaemic right lower leg. The right dorsalis pedis pulse is feeble. The right calf is tense and swollen. The intracompartmental pressure is 55 mmHg. Angiography shows no discontinuity of the arterial tree.

0

F Correct answer

Fasciotomy

This patient has the compartment syndrome. Urgent fasciotomy is indicated in order to save his right leg.

THEME: Principles of trial design and conduct

A Cross-over design B Detection bias C Double-blinding D Exclusion bias E Factorial design F Intention to treat G Minimisation H Null hypothesis I Parallel group design J Performance biasK PowerL Selection biasM Simple randomisationN Single blindingO Stratified randomisationP Subgroup analysisQ Type I errorR Type II error

The following are descriptions of terms used in the language of trial design. Please select the most appropriate descriptive term from the list above. The items may be used once, more than once, or not at all.

(Resources for this subject may be obtained from the BMJ book by Trisha Greenhalgh entitled How to read a paper.)

Scenario 1

In hypothesis testing: the term used to describe a situation in which we fail to reject the null hypothesis when a difference is really present.

0

R Correct answer

R – Type II error

This is the definition of a term used in the context of hypothesis testing. A type I error in contrast is one in which we reject the null hypothesis when a real difference is not present. These terms are most commonly cited in the context of study design where the probability of type I and II errors can be reduced by performing a prior power analysis in which the correct sample size is estimated on the basis of setting a and ß values which represent the probabilities that a type II and type I error will be committed. NB the null hypothesis is the cornerstone of hypotheticodeductive reasoning (Karl Popper) not a term describing a negative approach to research!

Scenario 2

An erroneous influence potentially effecting the conclusions of a trial caused by systematic differences in withdrawals from the trial.

0

D Correct answer

D – Exclusion bias

This is one of the four components of systematic bias (the others are in the list) that should be eliminated/minimised by good trial design and conduct. So-called ‘drop outs’ or exclusions from trials can occur for many reasons and can introduce bias quite easily since the tendency (even unintentially) is to exclude participants to favour the outcome of the trial. Where exclusions occur, this problem can be reduced by analysis on an ‘intention to treat’ basis (ie they are still included in the analysis).

Scenario 3

A method of allocation in comparative studies that provides treatment groups that are very closely similar for several variables.

0

G Correct answer

G – Minimisation

This is an alternative to simple randomisation (the commonest method used to reduce selection bias) when this might potentially introduce large differences in the characteristics of comparison groups within a trial. Other methods include stratified randomisation but this is usually used for single binary variables such as sex.

THEME: Miscellaneous disorders of the female breast 

A Absence of breast tissueB Accessory breast tissueC Cystosarcoma phylloidesD Idiopathic benign breast hypertrophyE LipomaF Lobular mastitisG Mondor’s diseaseH Peri-ductal mastitisI Sclerosing adenosisJ Traumatic fat necrosisK Tuberculosis of the breast

The following scenario describes presentation of a woman with breast pathology. From the list above, choose the single most appropriate diagnosis. Each item may be used once, more than once, or not at all.

 

Scenario 1

A 55-year-old woman complains of a 2-day history of pain in her right breast. The pain is described as localised to the lateral aspect of the breast and of sudden onset. She has no significant risk factors for breast cancer; however, she is extremely anxious. On examination she has a tender, subcutaneous, linear cord in the right breast. The overlying skin is acutely inflamed and becomes puckered on raising the right arm. Mammography and breast ultrasound are unremarkable.

0

G Correct answer

G – Mondor’s disease

In a patient complaining of sudden breast pain, the finding of a tender subcutaneous cord, attached to the skin, is pathognomic of Mondor’s disease. This rare condition is characterised by a sclerosing thrombophlebitis of the subcutaneous veins of the anterior chest wall, which give rise to the clinical findings. Imaging studies tend to be unremarkable. The condition is benign and self-limiting although a fibrous subcutaneous band may remain. Treatment is symptomatic with non-steroidal anti-inflammatories.

THEME: Knee injury

A Medial meniscal tearB Lateral meniscal tearC Anterior cruciate ligament rupture

D Posterior cruciate ligament ruptureE Medial collateral ligament tearF Lateral collateral ligament tearG Knee effusion

What is the injury in the following patients?

A history of twisting injury and pain at the antero-medial aspect of the joint is indicative of a medial meniscal injury. If bleeding has occurred into the joint space, the knee swells within 1–2 h. This is usually suggestive of a serious intra-articular damage such as an anterior cruciate ligament rupture. If the knee swells over a 12–24 h period, the swelling is more likely to be due to a simple effusion. As fluid collects in the knee, movement becomes more restricted and painful and only a few degrees of movement may be possible. The knee is usually held in 10 of flexion. In collateral ligament injuries, the pain is usually localised; however, no clear tenderness can be elicited in cruciate ligament injuries.

Scenario 1

A 25-year-old man sustains injury to his right knee during a football tackle. However, he continues to play albeit with severe discomfort. He presents to the emergency department after 24 h with a swollen, tender right knee. He is unable to straighten the knee fully and tenderness is elicited over the anterior medial aspect of the knee at the level of the joint line.

0

A Correct answer

Scenario 2

A 25-year-old man sustains severe injury to his left knee from an awkward fall while skiing. Swelling in this knee is noticed immediately. There is no localised joint line tenderness, but he is unable to fully extend this knee.

0

C Correct answer

THEME: Acid–base balance

pH PaCO2 HCO3-

A 7.20  3.1 11B 7.42 6.1 35C 7.56 3.0 30

For each of the patients described below, select the most likely set of blood gas measurements from the list of options above. Each option may be used once, more than once, or not at all.

Intestinal obstruction would lead to loss of fluid from the intravascular space and produce third-space loss. The presence of peritonitis and a grossly elevated WBC strongly suggest the presence of infarcted bowel. All these factors would produce a metabolic acidosis.

The history in the second case is suggestive of pulmonary emboli. Pulmonary emboli may occur at any stage after surgery, but most frequently after 5 days, and should be suspected in all cases of dyspnoea (earliest sign). The ensuing hyperventilation would reduce PCO2, shifting the Henderson–Hasselbach equation to the right (reducing H+), causing a respiratory alkalosis.

Patient 3 may not be taking deep enough breaths due to pain from his laparotomy, which would be exacerbated by COPD from his smoking. He will therefore have a respiratory acidosis due to retained carbon dioxide. Note there is a compensatory rise in the serum bicarbonate level.

Normal values:pH 7.35–7.45PaO2 >10.6 kPaPaCO2 4.7–6.0 kPaHCO3

–  24–30 mmol/l

 

Scenario 1

A 68-year-old woman with a history of abdominal pain with rebound tenderness and guarding for 72 hours. She exhibits signs of peritonitis and a WBC of 23 x 109/l.

0

A Correct answer

A – pH 7.20, PaCO2 3.1, HCO3- 11

Scenario 2

A 45-year-old woman who had a total abdominal hysterectomy for fibroids is now complaining of shortness of breath 7 days after surgery. There is no history of haemoptysis or pleuritic chest pain.

0

C Correct answer

C – pH 7.56 PaCO2 3.0, HCO3- 30

Scenario 3

A 75-year-old man had an uneventful abdominal aortic aneurysm (AAA) repair 10 days ago. He is known to smoke 30 cigarettes per day.

0

B Correct answer

B – pH 7.42 PaCO2 6.1, HCO3- 35

THEME: Nipple discharge

A Breast carcinomaB Epithelial hyperplasiaC Fibrocystic diseaseD Intraductal carcinomaE Mammary duct ectasiaF Mammary duct fistulaG Mammary intraductal papillomaH Peri-ductal mastitisI PhysiologicalJ Prolactinoma

The following scenarios describe patients presenting with nipple discharge. From the above list of possible causes, choose the most likely diagnosis. Each answer may be chosen once, more than once, or not at all.

Up to two-thirds of all normal non-lactating women produce fluid from the nipple if a small amount of negative-pressure is applied, and this varies from yellow to dark green. However, it is never blood-stained. Management is influenced by triple assessment, comprising clinical assessment, imaging and cytology. Haemo-Stix is an easy bedside test used to test for the presence of blood in discharge.

Scenario 1

A 55-year-old lady presents with a 2-month history of ‘cheesy’ nipple discharge. On examination her breast is tender, and her nipple is retracted. You are able to express discharge from multiple ducts, which tests positive with Haemo-Stix. Mammography on the left shows some coarse calcification behind the nipple.

0

E Correct answer

E – Mammary duct ectasia

Duct ectasia or duct dilatation can occur as part of the normal involution process of the breast. Symptomatic duct ectasia is benign and should therefore be considered as an aberration of this normal process. In the past, peri-ductal mastitis was thought to be the same disease process, but this has since been demonstrated as a separate pathological entity. Duct ectasia usually affects women between the ages of 50 and 60 years, and patients present as described in the scenario with nipple retraction and discharge. It is rarely frankly blood-stained and is often multiductal. The majority of patients have normal mammograms, although some radiographs can demonstrate duct thickening and coarse retro-alveolar calcification.

Scenario 2

A 60-year-old lady is re-called following a screening mammogram that demonstrates a dilated retro-areolar duct. She recalls a brief episode 3 months previously of blood-stained nipple discharge that you are able to reproduce clinically. Her breast examination is otherwise normal.

0

G Correct answer

G – Mammary intraductal papilloma

The lesion is benign but associated with an increased risk of developing an invasive breast carcinoma. Patients present clinically either with serous or bloody nipple discharge, or with a small sub-areolar tumour. Mammography may be abnormal as described although there may also be no lesion seen. Treatment involves microdochectomy, usually performed following the insertion of a fine probe into the affected duct while the patient is awake. A total duct excision may be performed if there are multiple ducts involved, but this is associated with a 20% risk of loss of nipple sensation.

Scenario 3

A 49-year-old man presents with a watery, occasionally purulent nipple discharge that has been worsening over the last 6 months. He is a smoker. On examination there is marked peri-areolar inflammation and a fluctuant area lateral to the affected nipple.

0

H Correct answer

H – Peri-ductal mastitis

This question highlights that men also suffer from benign breast disease. As mentioned earlier, peri-ductal mastitis has, in the past, been mistakenly confused with duct ectasia. Peri-ductal mastitis is characterised by peri-areolar inflammation, a non-lactating breast abscess, or a mammary duct fistula. Patients may also describe severe breast pain. An important distinguishing aetiological factor in peri-ductal mastitis is smoking; 90% of affected women are smokers compared with 38% in an unaffected agematched population. Treatment involves drainage of any underlying abscess, antibiotic treatment and, in recurrent cases, excision of the affected tissue. Care needs to be taken because drainage may result in fistula formation, and co-incidental malignancy needs to be excluded.

Scenario 4

A 45-year-old woman with a family history of breast cancer presents with a 3-week history of blood-stained nipple discharge. She reports that it is consistently produced from the same small area on her right nipple. On examination you concur, but otherwise there are no other notable findings. Fine-needle aspirate is reported as suspicious and mammography reveals fine microcalcification.

0

B Correct answer

B – Epithelial hyperplasia

Epithelial hyperplasia is described as an increase in the number of layers of lining epithelial cells in the terminal duct lobular unit. If there is evidence of atypia in the cells, then this is known as atypical hyperplasia, and is associated with a four to five times increased risk of breast cancer compared with the normal population. However, some mild epithelial hyperplasia is a normal variant seen in pre-menopausal breast tissue. Atypical hyperplasia is further divided into atypical lobular or ductal hyperplasia, dependent on histological findings and not on origins of the cells. Women with atypical ductal hyperplasia who have a first-degree relative with breast cancer have a 20–30% absolute risk of developing breast cancer over the following 20 years. Patients with atypical lobular hyperplasia are likely to develop cancer with an equal frequency in either breast, compared to patients with ductal hyperplasia, which tends to be confined to the breast in which it is biopsied. Although the answer to this scenario may have been breast carcinoma, this rarely presents with simply nipple discharge.

THEME: Endocrine

Treatment of thyrotoxicosisA PropranololB CarbimazoleC Radio-iodineD Subtotal thyroidectomyE Total thyroidectomy

Thyrotoxicosis is managed conservatively in the first instance. Propranolol, a  blocker, is good for reducing anxiety and treatment of cardiac arrhythmias such as AF. Care must be taken when prescribing carbimazole because blood dyscrasias occur in 2% of cases.

Scenario 1

A pregnant woman.

0

B Correct answer

Scenario 2

A teenager.

0

B Correct answer

Scenario 3

A 65-year-old individual with cerebrovascular accident (CVA)/atrial fibrillation (AF).

0

A Correct answer

Scenario 4

Failed medical treatment.

0

D Correct answer

THEME: Interpretation of pupil size

A Oculomotor nerve compression

B Horner's syndrome

C Drugs

D Pontine lesion

E Optic nerve injury

F Metabolic encephalopathy

G None of the above

For each of the operative scenarios listed below, select the most likely cause of the pupillary findings. Each option may be used once, more than once, or not at all.

 

Scenario 1

A 75-year-old male smoker is admitted short of breath and complaining of right-sided chest pain. He has a mass in the apex of his right lung. He is given intravenous

morphine for pain relief. On examination you note that he has mild right-sided ptosis and miosis.

0

B Correct answer

Horner’s syndrome is unilateral miosis, partial ptosis and anhydrosis. It is caused by disruption of the cervical sympathetics, in this case by an apical lung tumour.

Scenario 2

An 18-year-old female is brought in to the emergency department. She has been found collapsed at the back of a nightclub. There is no evidence of a head injury. She has a reduced Glasgow Coma Scale. Both pupils are constricted and it is difficult to assess whether they are responsive to light.

0

C Correct answer

Causes of bilaterally small pupils include drugs (opiates), a destructive lesion of the pons and a metabolic encephalopathy. In this case the most likely cause is illicit drug use.

Scenario 3

During the secondary survey of a severely head-injured patient, you find mild bilateral dilatation and a sluggish papillary response to light.

0

A Correct answer

An early sign of temporal lobe herniation is mild papillary dilatation and a sluggish response to light. This is caused by mild compression of the oculomotor nerve (cranial nerve III). As herniation worsens there is further dilatation, ptosis and paralysis of the ocular muscles innervated by CN III.

Scenario 4

A workman is injured in an explosion on a building site. He has shrapnel wounds to his head and chest. His pupils seem to be equal. However, the right pupil does not react directly to light but does react to light in the opposite eye.

0

E Correct answer

This patient has sustained a penetrating injury to his right optic nerve. This leads to a pupil that is cross reactive (Marcus–Gunn pupil) and may be dilated or normal in size.

THEME: SWALLOWING DIFFICULTIES

A Candida oesophagitisB AchalasiaC Oesophageal carcinomaD Perforated oesophagusE Oesophageal strictureF Hiatus herniaG Pharyngeal pouch

For each of the patients described below, select the single most likely diagnosis from the options listed above. Each option may be used once, more than once, or not at all.

 

Scenario 1

A patient presents with chronic obstructive pulmonary disease (COPD). He is on steroids, with post-operative difficulties and pain on swallowing.

0

A Correct answer

A patient on steroids who complains of difficulty on swallowing is most likely to have candidal oesophagitis. This can be treated with nystatin.

Scenario 2

A patient presents with vomiting and mediastinal gas on CXR.

0

D Correct answer

Vomiting and mediastinal gas on CXR is most likely to be due to perforated oesophagus. This is known as Boerhaave’s syndrome.

Scenario 3

A 28-year-old woman presents with halitosis, regurgitation and flatulence.

0

F Correct answer

A 28-year-old with halitosis, regurgitation and flatulence is most likely to have a hiatus hernia. Halitosis and regurgitation is also found in pharyngeal pouch, but this is more commonly found in elderly patients.

Scenario 4

A 58-year-old man suffers painful dysphagia 5 days following laparotomy for a ruptured appendix for which he received post-operative cefuroxime and metronidazole.

0

A Correct answer

The clinical case scenario of this 58-year-old man is most likely due to candidal oesophagitis. Antibiotics can also lead to fungal infections.

Scenario 5

A 77-year-old man is taking a soft diet due to increasing difficulty in swallowing. He describes a lifelong history of indigestion and is now losing weight.

0

C Correct answer

In the clinical case scenario of the 77-year-old man, one should suspect an oesophageal carcinoma.

THEME: HepatobiliaryThe best treatmentA Medical therapyB Elective cholecystectomyC Emergency cholecystectomyD Endoscopic retrograde cholangiopancreatography (ERCP)

If patients are fit enough and have symptoms from gallstones, a cholecystectomy should be performed. Almost 90% of all cholecystectomies are performed laparoscopically in the UK. If patients are too elderly or unfit for general anaesthesia, an ERCP may be considered as an alternative.

Scenario 1

An old woman not fit for surgery with stones in the common bile duct plus jaundice.

0

D Correct answer

Scenario 2

A young woman with fat intolerance, right upper quadrant pain and gallstones.

0

B Correct answer

THEME: Paediatric newborn gastrointestinal disorders

A Biliary atresia B Duodenal atresia C Hirschsprung’s disease D IntussusceptionE Meconium ileusF Necrotising enterocolitisG Pyloric stenosis

For each of the patients described below, select the most likely diagnosis from the list of options above. Each option may be used once, more than once, or not at all.

A child with a scaphoid abdomen and bilious vomiting should be suspected of having duodenal atresia. Symptoms usually occur in the first few hours of life. The diagnosis may be made by injecting 30 ml of air via a nasogastric tube and taking a radiograph. This is not to be confused with the persistent vomiting of hypertrophic pyloric stenosis, which is not bile stained and presents at 6 weeks.

A child with an abdominal mass, distension, passing meconium for 3 days and now passing blood, should be suspected of having an intussusception. It is only late in the disease that bleeding per rectum becomes a feature, suggesting mucosal necrosis.

Meconium ileus tends to present with a distended abdomen and bilious vomiting with no passage of meconium. The rectum is characteristically empty. Distal small bowel obstruction, secondary to abnormally bulky viscid meconium is seen in 1 : 15 000 newborns. Ninety per cent of these infants will have cystic fibrosis.

Hirschsprung’s disease is the most common cause of intestinal obstruction in a newborn and affects 1 : 5000 children. Typically, there is delay in passing meconium beyond the first 24 hours of life with abdominal distension and bilious vomiting. Patients with Hirschsprung’s disease may present after the neonatal period with chronic constipation or partial large bowel obstruction.

Scenario 1

A 3-day-old child presents with a scaphoid abdomen, and is unable to feed with bilious vomiting.

0

B Correct answer

B – Duodenal atresia

Scenario 2

A 10-day-old child presents with a right abdominal mass and distension, with a history of passing meconium for only three days with the help of suppositories. He is now passing blood per rectum.

0

D Correct answer

D – Intussusception

Scenario 3

A baby presents needing enemas to open its bowels owing to a 24-hour delay in passing meconium.

0

C Correct answer

C – Hirschsprung’s disease

THEME: Bone and soft tissue tumoursA Bone metastases B Chondroma C Chondrosarcoma D Ewing’s sarcoma E Fibroma F Fibrosarcoma G Leiomyoma H Leiomyosarcoma I LipomaJ LiposarcomaK OsteomaL OsteoblastomaM OsteochondromaN Osteosarcoma (osteogenic sarcoma)O Osteoid osteomaP Simple bone cyst

The following patients all present with bone or soft tissue tumours. From the list above, select the most likely diagnosis. The items may be used once, more than once, or not at all.

Primary tumours of bone and soft tissue are uncommon. They usually present with unremitting pain, swelling and loss of function. They may also present with a pathological fracture, a joint effusion or systemic symptoms when there is metastatic spread. In the diagnosis of bone tumours, the frequency of various tumours, the patient’s age, the bone affected and the location of the lesion within the bone, and its radiological appearances are all important.

Scenario 1

A 15-year-old boy attends The Emergency Department. He describes pain affecting his left femur that was initially an ache but that has now become severe and constant. In addition, he reports generalised malaise and a persistent cough. On examination, he has an antalgic gait and there is asymmetrical swelling affecting the distal left femur. Radiology reveals breach of the periosteum, which is elevated, and a ‘sunray’ appearance affecting the distal femur.

0

N Correct answer

N – Osteosarcoma (osteogenic sarcoma)

This is one of the commonest primary malignant bone tumours affecting the young with a slight male predominance. There is also a second peak in incidence in the elderly related to Paget’s disease. The long bone metaphyses are usually affected, most commonly around the knee. Bloodborne metastases develop early and spread to the lungs and other parts of the skeleton. Cortical penetration with peri-osteal elevation gives rise to ‘Codman’s triangle’ on X-ray. In addition, a characteristic sunray appearance may be evident on a plain radiograph as a result of new bone formation. In terms of histology, some may be largely fibroblastic, others are osteoblastic (with pleomorphic cells), some comprise chondroid osteoblasts, and they may even be highly vascular (telangiectatic). All form osteoid and/or bone-incorporating malignant cells. Computed tomography and magnetic resonance scanning are important in staging. Treatment involves chemotherapy combined with wide surgical excision (with limb salvage if possible). Five-year survival rates of 60% can be expected in those without distant metastases.

Scenario 2

A 51-year-old woman is referred urgently by her general practitioner with ‘a right upper quadrant mass, which he suspects is malignant’. On examination, there is no evidence of an abdominal mass but there is a tender swelling arising from the right costal margin. An X-ray of the lesion reveals a localised area of bone destruction with mottled appearances, affecting the ninth rib near the costal margin.

0

C Correct answer

C – Chondrosarcoma

This is a malignant tumour of cartilage that affects the flat bones, vertebrae, girdles and the proximal limb bones in middle-aged patients. X-ray images may be diagnostic and reveal localised bone destruction, punctuated by mottled densities from calcification or ossification, as in the case described. Histological differentiation from enchondroma and osteosarcoma may be difficult. Treatment is similar to that for osteosarcomas, and 5-year survival rates are approximately 50%.

Scenario 3

A 12-year-old girl is referred with pain and swelling affecting her left lower leg. On examination, there is a tender, irregular swelling arising from the mid-tibia. X-ray reveals a destructive lesion, associated with a soft tissue mass, and peri-osteal ‘onion-skinning’. Biopsy demonstrates the presence of sheets of ‘small round cells’.

0

D Correct answer

D – Ewing’s sarcoma

This aggressive tumour is unusual in that it often affects the mid-diaphysis (shaft) of the bone, typically in children and adolescents. It is slightly less prevalent than osteosarcoma. It may masquerade as an osteomyelitis. Metastases arise in the liver, lung and other bones. Radiography reveals a characteristic ‘onion-skin’ appearance of the periosteum. Survival rates of 60% can be achieved at 5 years with the use of combined surgery, radiotherapy and chemotherapy.

THEME: 5-Year survival rates of tumours

A < 5% B 25% C > 95% D 5–10%E 50–60%F 70–75%G 90–95%

For each of the clinical scenarios listed below, select the most likely 5-year survival rate from the above list. Each option may be used once, more than once, or not at all.

The overall 5-year survival rates of patients with carcinoid tumours depend on the site of the primary. Those of the appendix have an approximate 98% 5-year survival; those of the rectum an 85% survival; and those of the small bowel have the lowest survival rate.

The 5-year survival rates (Duke’s A, B and C rectal cancers) have improved from the easily memorised ‘90%, 60%, and 30%’, respectively, to 92% (A), 71% (B), 40% (C1) and 26% (C2).Oesophageal and pancreatic cancers have a grave prognosis, and surgical cure is uncommon. Metastatic prostatic cancer can frequently be temporarily controlled by hormonal treatment.

Scenario 1

Carcinoid of the appendix

0

C Correct answer

C – > 95%

Scenario 2

Duke’s A rectal cancer

0

G Correct answer

G – 90–95%

Scenario 3

Oesophageal cancer

0

D Correct answer

D – 5–10%

Scenario 4

Pancreatic carcinoma

0

A Correct answer

A – < 5%

Scenario 5

Metastatic prostatic cancer

0

B Correct answer

B – 25%

Scenario 6

Duke’s B rectal cancer

0

F Correct answer

F – 70–75%

THEME: Common carcinogens A Arsenic B Asbestos C Aspergillus flavus D Benzol/ benzene type 1E ß-Naphthylamine F Cyclophosphamide G Diethyl stilboestrol H Epstein–Barr virus I Hardwood sawdust J Hepatitis B virus K Human immunodeficiency virusL Human papillomavirusM Human T-cell leukaemia virusN NickelO Nitrates/nitrites

P Nitrosamines/nitrosamidesQ SchistosomiasisR Ultraviolet lightS Vinyl chloride monomer 

The following scenarios describe patients who have had exposure to a carcinogen and subsequently developed a carcinoma. From the above list, choose the most appropriate carcinogen. Each item may be used once, more than once, or not at all.

 

Scenario 1

A 70-year-old man presents with haematuria. Cystoscopy demonstrates a transitional cell carcinoma of the bladder. He had been in the paint industry all his working life.

0

E Correct answer

E – ß-Naphthylamine

ß-Naphthylamine is used in the paint and dye industry and is not intrinsically carcinogenic (a remote carcinogen), but it undergoes conjugation in the liver to create a water-soluble carcinogen (a proximate carcinogen). This conjugated carcinogen is excreted by the kidneys and stored in the bladder. Here, micro-organisms secrete glucuronidase, which releases the ultimate carcinogen into the bladder. This, over a period of exposure, can result in transitional cell carcinoma.

Scenario 2

A 47-year-old African immigrant presents with hepatomegaly. Imaging reveals a large solitary hepatic lesion. He tells you that he was a crop farmer growing cereal.

0

C Correct answer

C – Aspergillus flavus

Aspergillus flavus produces Aflatoxin B1, which is a potent hepatocarcinogen in animals and is believed to be a factor in the high incidence of hepatocellular carcinoma in Africa (although you might not be blamed for choosing hepatitis B

virus). Aspergillus flavus is a fungus that grows on grains and peanuts. Repeated ingestion of infected food is a risk factor.

Scenario 3

A 55-year-old man who stayed frequently in Egypt complains of haematuria. He also has suffered in the past with recurrent renal and bladder stones.

0

Q Correct answer

Q – Schistosomiasis

This disease is caused by Schistosoma haematobium, a trematode fluke that lives in freshwater snails and parasitically in the human bladder. Schistosoma are found most commonly in Latin America, Africa and the Middle East. The eggs produced (up to hundreds a day) trigger granuloma formation which cause haematuria, cystitis, stenoses of the ureters and stone formation. The chronic inflammation induces squamous cell carcinoma of the bladder. Despite these urological manifestations, mortality is more commonly the result of portal vein granuloma formation with subsequent portal hypertension.

Scenario 4

A 30-year-old woman presents with vaginal bleeding. A recent cervical smear shows high-grade cervical dysplasia.

0

L Correct answer

L – Human papillomavirus

Human papillomavirus (HPV) has been implicated as a promoter of cervical carcinoma; HPV types 6 and 11 predominate in lower grade dysplasias (cervical intraepithelial neoplasia – CIN 1), while higher-grade dysplasias (CIN 2 and CIN 3) tend to be associated with HPV types 16, 31 and 33. Other co-factors include infection with herpes simplex virus type 1 and tobacco use. The CIN stages represent carcinoma in situ and up to 70% of women in whom this is left untreated will progress to developing invasive cervical carcinoma.

THEME: Distal radial fracture management

A Closed reduction and plaster of paris (POP) cast

B POP cast without manipulationC Intramedullary nailingD Plate and screwsE K-wiresF External fixation

Select the most appropriate method of management for each of the patients below. Each option may be used once, more than once, or not at all.

 

Scenario 1

A 76-year-old lady fell on her right outstretched arm sustaining a non-comminuted Colles’ fracture with 20 degrees of dorsal tilt.

0

A Correct answer

Any dorsal tilt in Colles’ fracture will need closed reduction if not comminuted to restore function.

Scenario 2

A 25-year-old male fell on the right outstretched hand while playing football sustaining a non-comminuted distal radial fracture with volar tilt.

0

D Correct answer

Smith fracture needs ORIF(open reduction and internal fixation) using a buttress plate because the distal fragment is unstable.

Scenario 3

A 55-year-old lady gets involved in a road traffic accident sustaining an open comminuted distal radial fracture with dorsal displacement.

0

F Correct answer

Ex-Fix will ease care of the wound which should not be closed as it may need debridement later; otherwise internal fixation will run the risk of implant infection and failure.

Scenario 4

A 10-year-old girl falls on the outstretched arm sustaining a buckled fracture of the distal radius. No angulation is evident on X-ray.

0

B Correct answer

Buckled and greenstick fractures are not manipulated if there is no angulation or tilt. Some surgeons don’t even manipulate it if the age is very young and there is no CLINICAL deformity despite a few degrees of tilt or angulation on the X-rays.

THEME: Pathologies of the femoro-inguinal region

A Psoas abscess B Obturator hernia C Femoral aneurysm D Strangulated femoral hernia E Enlargement of psoas bursaF False aneurysm of the femoral arteryG Local abscessH Strangulated inguinal herniaI Infected sebaceous cystJ Saphena varix

For each of the clinical scenarios listed below, select the lesion most likely to occur in that scenario from the above list. Each option may be used once, more than once, or not at all.

 

Scenario 1

A 60-year-old lady presents to the A&E with a temperature of 37.6°C and vomiting of 48 hours duration. On examination, a localised swelling below and lateral to the right

pubic tubercle is noticed. The swelling is tender and irreducible with no cough impulse.

0

D Correct answer

D – Strangulated femoral hernia

Femoral hernia is due to the protrusion of extra-peritoneal tissue and some abdominal contents through the femoral canal. It is more common in females (two and a half times) as the inguinal ligament makes a wider angle with the pubis. Femoral canal is bounded supero-anteriorly by the inguinal ligament, infero-posteriorly by the pubic ramus and pectineus muscle, medially by lacunar ligament and laterally by the femoral vein. Femoral hernia is differentiated from inguinal hernia as it lies lateral and inferior to pubic tubercle, while the inguinal hernia lies medial and above the pubic tubercle. Strangulated hernia is characterised by pain, irreducibility, and absent cough impulse. Necrosis of the involved viscus ensues with progressive loss of blood supply and this may lead to local, and then systemic sepsis, if untreated. The patients may present with general ill-health, vomiting and constipation. Associated pyrexia may be present if the infection becomes systemic.

Scenario 2

A 46-year-old lady presents with a soft, non-tender swelling over the medial side of her right thigh below and lateral to the pubic tubercle. The swelling has got a bluish tinge and disappears on lying down. She is well systemically but has got bilateral varicose veins.

0

J Correct answer

J – Saphena varix

Saphena varix is a saccular enlargement of the vein, usually seen at the proximal part of the long saphenous venous system. This is commonly seen in patients with varicose veins. The patients present with a swelling over the medial part of the upper thigh (where the long saphenous vein enters the femoral vein piercing the cribriform fascia) and the swelling often has a blue tinge. It disappears on lying down due to the emptying of the vein into the deeper system (femoral vein). There might be a palpable thrill when the patient coughs and an impulse is felt over the varix when the vein is tapped from below (Schwart’s test).

Scenario 3

A 35-year-old man of Asian origin presents with a painless, fluctuant swelling over the upper medial side of his left thigh. He has been previously treated for tuberculosis. His ESR is 113. He has also got some tenderness over the lower spine and X-ray reveals some opacity in this region.

0

A Correct answer

A – Psoas abscess

Psoas abscess is usually due to a cold abscess tracking down towards the inguinal region from the spine along the psoas muscle. This is usually due to tuberculosis affecting the spine (Pott’s disease). The abscess is usually fluctuant, painless and not warm (hence the name cold abscess which is characteristic of tuberculosis). Often there may be cross fluctuation (above the inguinal ligament) due to an associated iliac abscess. The diagnosis is made from the history and clinical examination. It can be confirmed either by radiograph, ultrasound or CT scanning of the spine. Associated organ involvement should be actively looked for such as lung involvement, for example.

Scenario 4

A 28-year-old IV drug abuser presents with a mildly tender, pulsatile swelling over his right femoral triangle. He gives a history of considerable bleeding from the wound before the swelling developed. He is apyrexial and systemically well.

0

F Correct answer

F – False aneurysm of the femoral artery

False aneurysm of the femoral artery occurs as a result of laceration or injury to the arterial wall (usually tunica adventitia). The injury leads to local haematoma which becomes contained in the surrounding normal tissue. There is often a history of considerable primary haemorrhage from the wound which usually settles with sustained pressure. This is common with IV drug abusers as they may accidentally puncture the femoral artery when attempting to inject the drug into the adjacent femoral vein as the artery and the vein lies in close proximity in the femoral triangle. Distal pulsation in the affected limb is usually maintained. There may be local bruit over the involved arterial region and the appearance of the expansile pulsation in the region usually clinches the diagnosis.

THEME: Venous drainage of the suprarenal gland

A Splenic veinB Left renal veinC Right renal veinD Inferior mesenteric veinE Inferior vena cava

Pick the most appropriate option from the above list. Each option may be used once only, more than once or not at all.

The right suprarenal vein enters the inferior vena cava and the left vein is longer and enters the left renal vein.

Scenario 1

What is the venous drainage for the right suprarenal gland?

0

E Correct answer

E - Inferior vena cava

Scenario 2

What is the venous drainage for the left suprarenal gland?

0

B Correct answer

B - Left renal vein

THEME: Management of gallstone disease A Endoscopic Retrograde Cholangio-Pancreatogram (ERCP) and          sphincterotomyB Early laparoscopic cholecystectomy +/- pre-operative cholangiographyC Interval laparoscopic cholecystectomy +/- pre-operative          cholangiographyD Laparoscopic cholecystectomy + common bile duct explorationE Laparotomy with cholecystotomy/drainageF Magnetic resonance cholangiopancreatogram (MRCP)G Open cholecystectomy + common bile duct exploration

H Non-surgical treatment, eg chemical dissolution therapyI Percutaneous (radiological) gallbladder drainage 

The following scenarios describe patients suffering from a complication of gallstone disease. From the above list, choose the single most appropriate definitive management option. Each item may be used once, more than once, or not at all.

The management of symptomatic gallstone disease has changed greatly over the last 20 years. It is an area of contention in many respects, and many algorithms exist (especially for the suspected bile duct stone) all of which have their merits. The final decision probably rests with the experience of the surgeon and the equipment available. The MRCS candidate might be advised to select the simplest and safest answer where doubt exists. These are given below.

Scenario 1

A 45-year-old woman presents with a 2-day history of right upper quadrant pain, rigors, nausea and vomiting. She is febrile and her sclerae are noted to be yellow. Abdominal examination reveals a tender right hypochondrium. Liver function tests: bilirubin 29 µmol/l, alkaline phosphatase 450, aspartate aminotransferase 25 iu/litre. Ultrasound demonstrates a common bile duct diameter of 8 mm with multiple stones present.

0

A Correct answer

A – ERCP and sphincterotomy

This lady is suffering from cholangitis as defined by Charcot’s triad. The common bile duct is dilated (> 6–7 mm). She requires urgent drainage. This should be performed using ERCP with a laparoscopic cholecystectomy deferred to a later date. Failure to clear the duct by simple measures should prompt the use of a stent or, if this fails, transhepatic drainage (percutaneous transhepatic cholangiogram). Open surgical drainage is rarely required.

Scenario 2

A 35-year-old woman presents with worsening pain in her right upper quadrant associated with nausea and vomiting. She is neither clinically nor biochemically

jaundiced but is febrile. An ultrasound reveals gallstones, thickening of the gallbladder wall and a common bile duct diameter of 3 mm.

0

B Correct answer

B – Early laparoscopic cholecystectomy +/- pre-operative cholangiography

This lady has acute cholecystitis with no evidence of common bile duct stones. The main choice lies between early (on the index admission) or interval cholecystectomy +/- pre-operative cholangiography. Opinion is divided but, increasingly, surgery is being advocated on the index admission. This is in the acceptance that although the rate of conversion to open surgery is high (20% approximately), it is similarly high at a later date and further complications, including further episodes of inflammation, occur with regularity while on the waiting list.

Scenario 3

A 79-year-old man presents with a 5-day history of progressive right upper quadrant pain. On examination, he is clearly septic. Examination reveals a very tender mass in the right upper quadrant. White cell count 21 x 109/litre.

0

I Correct answer

I – Percutaneous (radiological) gallbladder drainage

In a fit person, empyema of the gallbladder (as presented here) might be managed by emergency open cholecystectomy. Before the advent of interventional radiology, if experience was not available, laparotomy could be performed with open drainage. The best treatment, however, in this case would be radiological drainage with antibiotics/resuscitative measures, which will hopefully rectify the situation without recourse to life threatening surgery.

THEME: Epistaxis

A Chemical irritationB Foreign bodyC HaemophiliaD HypertensionE IatrogenicF IdiopathicG Malignant neoplasmH Hereditary haemorrhagic telangectasia (Osler’s disease)I Pyogenic granuloma

J RhinitisK ThrombocytopaeniaL TraumaM Wegener’s granuloma

The following patients have all presented with epistaxis. Please select the most appropriate diagnosis from the above list. The items may be used once, more than once, or not at all.

The commonest causes of epistaxis are: idiopathic (nose picking), external trauma and rhinitis (allergic and infective). Other causes may also be local or secondary to systemic disease (especially blood dyscrasias). Iatrogenic causes include anticoagulant therapy and nasogastric tubes.

Scenario 1

A 2-year-old child presents with bleeding from one side of the nose. His mother had noticed a foul-smelling discharge from the nose on that side for some months. This had only temporarily responded to courses of antibiotics. Examination of the nostrils shows an inflamed mucous membrane and a blood-stained mucopurulent discharge.

0

B Correct answer

B – Foreign body

This may present with bleeding from the nose in the presence of longstanding inflammation. The history of a foul-smelling nasal discharge and the age of the child (foreign bodies in the nose are commonest in children aged 2–3) should alert one to the diagnosis. Treatment is by removal under general anaesthesia once inflammation is established as in this case. If the problem is identified early, various manoeuvres can be attempted in The Emergency Department to blow out the offending foreign body.

Scenario 2

A 25-year-old man presents with a history of chronic sinusitis and epistaxis over the past 3 years. On rhinoscopy he has nasal crusting with a small septal defect. On oral examination there is quite marked gingivitis and tooth decay.

0

A Correct answer

A – Chemical irritation

In this case, the patient is likely to be a regular user of cocaine (now one of the most common causes of recurrent epistaxis). The diagnosis is strongly suggested by the septal defect, dental problems and sinus problems, especially in a young adult.

THEME: Diseases of the stomach

A Carcinoid tumourB Krukenberg tumoursC AdenocarcinomaD LymphomaE Gastric ulcerationF LeiomyomaG Chronic gastritis

For each of the following statements, select the most likely diagnosis from the above list. Each option may be used once, more than once, or not at all.

Autoimmune chronic gastritis is commonly associated with macrocytic anaemia due to vitamin B12 deficiency (pernicious anaemia). This is also a risk factor for carcinoma. Acute ulceration is seen with head injury (Cushing’s ulcer) and burns (Curling’s ulcer). Carcinoid tumours give rise to facial flushing, diarrhoea and right-sided valvular disease due to collagen deposition.

Scenario 1

Commonly associated with vitamin B12 deficiency

0

G Correct answer

Scenario 2

May cause pulmonary valve stenosis

0

A Correct answer

Scenario 3

Transcoelomic spread of tumour to the ovaries

0

B Correct answer

Scenario 4

May present with Troisier’s sign

0

C Correct answer

Scenario 5

Associated with head injury

0

E Correct answer

THEME: Surgical sutures

A SilkB VicrylC Cat gutD NylonE Steel wireF LinenG Dacron

For each of the statements below select the most likely answer from the list above. Each option may be used once, more than once or not at all.

 

Scenario 1

It is made of only one filament

0

D Correct answer

Nylon is a monofilament.

Scenario 2

It is least likely to cause tissue irritation

0

D Correct answer

All sutures are foreign bodies and therefore irritating to tissues; the synthetic sutures are less irritating, with monofilament being the least irritating.

Scenario 3

It is a synthetic absorbable suture for subcuticular skin closure

0

B Correct answer

Vicryl is absorbable and is used for subcuticular skin closure, eliminating the need to remove any sutures postoperatively.

Scenario 4

Its strength is unchanged with time

0

E Correct answer

All sutures will eventually lose their strength even the so called non-absorbable materials, except steel wire.

Scenario 5

It is the quickest to be absorbed

0

C Correct answer

Scenario 6

It is used for abdominal wall closure

0

D Correct answer

THEME: Knee injuries

A Medical meniscusB Lateral meniscusC Medial collateral ligamentD Lateral collateral ligamentE Patellar fractureF Anterior cruciate ligamentG Posterior cruciate ligament

The history in the young footballer is most suggestive of a medial collateral ligament injury because there is a valgus deformity and tenderness above the joint line. Tenderness over the joint line is suggestive of meniscal injury. The posterior cruciate ligament prevents the femur from sliding forwards off the tibial plateau. In the weight-bearing flexed knee, it is the only stabilising factor for the femur and its attached quadriceps.

In walking downstairs the upper knee is flexed and weight bearing whereas the lower knee is straight. The anterior cruciate ligament prevents backward displacement of the femur on the tibial plateau and limits extension of the lateral femoral condyle.

Immediate haemarthrosis is indicative of cruciate ligament rupture or fracture.

Scenario 1

A young footballer injured his right knee in a tackle, and had swelling of the knee a few days later. He presented to A&E with tenderness in an area 2–3 cm above the joint line on the medial aspect. There is increased valgus deformity on examination.

0

C Correct answer

Scenario 2

A 60-year-old patient developed a painful left knee after a fall. On examination, swelling on the left knee was seen in the orthopaedic clinic a few days later. Flexion views of both knees showed that the left tibial plateau lay more posteriorly than that on the right.

0

F Correct answer

Scenario 3

A patient involved in a road traffic accident (RTA) hit his knee on the dashboard, producing an immediate haemarthrosis. The X-ray showed no bone injury, but he later experienced difficulty going downstairs.

0

G Correct answer

THEME: Bone and connective tissue tumours A Chondrosarcoma B Ewing’s sarcoma C Fibrosarcoma D LeiomyosarcomaE Malignant giant cell tumour F OsteochondromaG OsteoclastomaH Osteoid osteomaI OsteosarcomaJ Rhabdomyosarcoma 

For each of the following statements, select the most likely cause of pain/swelling from the above list. Each option may be used once, more than once, or not at all.

 

Scenario 1

A 65-year-old presents with a swelling and pain over her left proximal humerus. The pain is worse at night. She also gives a history of weight loss. She has a raised ESR and she is undergoing treatment for Paget’s disease.

0

I Correct answer

I – Osteosarcoma

Osteosarcoma, the most frequently encountered malignant lesion of bone, is characterised by the direct formation of bone or osteoid tissue by a sarcomatous stroma. It typically affects the knee and the proximal humerus in the metaphyseal region. The incidence is highest in the 10–25-years age group. Secondary osteosarcoma, however, may arise in the bones of the elderly affected by Paget’s disease (in approximately 10% of patients affected for > 10 years) or after irradiation. The characteristic symptoms of pain, local tenderness, a soft tissue mass and a decreased function may be present for variable periods of time. On examination, the affected part is swollen and the overlying skin may be shiny and warm. The lump is tender and has irregular edges. The ESR may be raised. X-ray shows bone destruction and new bone formation, often with marked periosteal elevation (‘Sunray spiculation’ and ‘Codman’s triangle’, respectively). Surgical excision is the treatment of choice.

Scenario 2

A 13-year-old adolescent is brought by his parents to the GP with loss of weight, pain and fever. On examination, a soft but tender, ill-defined mass is palpable over his mid-thigh region.

0

B Correct answer

B – Ewing’s sarcoma

Ewing’s sarcoma is a malignant tumour arising from the vascular endothelium of the bone marrow. The tumour is common in the 10–20-years age group and occurs in the

diaphysis of the long bones. Clinical features include pain and swelling; the lump is warm and tender, with ill-defined edges. The ESR may be elevated, thus spuriously suggesting an inflammatory or infective cause such as osteomyelitis, although osteomyelitis usually affects the metaphyseal region in children. X-rays often show a large soft-tissue mass with concentric layers of new bone formation – known as ‘onion-peel’ sign. Treatment includes chemotherapy and surgical excision.

Scenario 3

A 23-year-old presents with mild discomfort and a lump over his right knee. He accidentally discovered the lump one week ago. On examination, it is bony hard and non-tender. He is systemically well.

0

F Correct answer

F – Osteochondroma

Osteochondroma (cartilage-capped exostosis) is the most common benign tumour of the bone. The usual site for the tumour is the metaphysis of the long bones. The lesion may be single or multiple (hereditary multiple exostoses). The usual history is of a lump that is discovered accidentally. The lump is bony hard and non-tender. X-ray reveals a well-defined swelling; however, the swelling looks smaller than it feels because of the invisible cartilaginous cap.

Scenario 4

A 28-year-old lady presents with weight loss, fever and a swelling over her right knee of a few weeks duration. Movements of her knee are severely restricted. X-ray reveals thinning of the cortex and a fracture of the distal femur. An extraosseous soft tissue mass is seen on MRI scan.

0

G Correct answer

G – Osteoclastoma

Osteoclastoma (giant cell tumour) is an uncommon, aggressive, locally destructive lesion seen in the metaphyseo-epiphyseal region of long bones. It frequently occurs in young adults 20–40 years of age and is more common in women. The principal sites are the distal femur, proximal tibia, proximal fibula, distal radius and proximal humerus. On examination, a vague swelling is felt at the end of long bones and the neighbouring joint is often inflamed. Although < 5% of these tumours metastasise, the

lesions are extremely destructive, sometimes locally resulting in pathological fractures (approximately 10% of cases) as seen in this patient. X-ray shows thinning of the cortex giving it an expanded appearance – the characteristic ‘soap bubble’ appearance. Radiotherapy is the treatment of choice.

THEME: Basic statistics (taxonomy) A Analysis of variance (ANOVA) B 2 test C Correlation D False negatives E False positives F Likelihood ratio G Negative predictive valueH Positive predictive valueI Power analysisJ Regression analysisK SensitivityL SpecificityM t-tests

The following descriptions all refer to basic statistical methods that are commonly employed in medical research. Please select the most appropriate term from the above list. The items may be used once, more than once, or not at all.

(Basic resources for this subject may be obtained from the chapter by Hugh Dudley in the Clinical Surgery in General RCS Course manual or, for the more avid student, from a book entitled Practical Statistics for Medical Research by Douglas G Altman.)

Scenario 1

A method of statistical analysis used for hypothesis testing where we wish to compare proportions of categorical data.

0

B Correct answer

B – 2 test

This is the correct test for categorical data, such as male/female, where we wish to establish whether there is a significant difference in proportions between two or more groups (the Fisher’s exact test is similar but utilised for only 2 x 2-cell tables with small numbers in each cell). In contrast, t-tests, such as the Student’s t-test

(parametric data) and the Mann–Whitney U-test (non-parametric data) are used for comparing continuous numerical data.

Scenario 2

In the assessment of a diagnostic test, the proportion of patients with negative test results who are correctly diagnosed.

0

G Correct answer

G – The negative predictive value

The proportion of patients with negative test results who are correctly diagnosed is the negative predictive value. In contrast, the proportion of patients with positive test results who are correctly diagnosed is the positive predictive value. Unlike specificity and sensitivity, the positive and negative predictive values give a direct assessment of the usefulness of a test in practice. However in addition, unlike sensitivity and specificity, the positive and negative predictive values are strongly affected by prevalence (ie the proportion of patients with the abnormality). The positive predictive value increases with increasing prevalence and the negative predictive value decreases.

Scenario 3

This may be calculated to indicate the value of a test for increasing certainty about a positive diagnosis. It is numerically equal to the sensitivity/(1 – specificity).

0

F Correct answer

F – Likelihood ratio

For any test result, one can compare the probability of getting that result if the patient truly had the disease with the corresponding probability if they were healthy. The ratio of these probabilities is the likelihood ratio, which can be considered to indicate the value of a test for increasing certainty about a positive diagnosis. It is numerically equal to the sensitivity / (1 – specificity).

THEME: Thoracic incisions

A Median sternotomyB Posterolateral thoracotomy

C Clamshell thoracotomyD Anterior thoracotomyFor each of the following situations select the most useful surgical incision. Each option may be used once, more than once, or not at all.

 

Scenario 1

Aortic transection at the junction of the aortic arch with descending thoracic aorta

0

B Correct answer

The junction of the aortic arch with the descending thoracic aorta is the commonest site of aortic transection, often after a deceleration injury. The aorta is found in the posterior mediastinum and is best approached via a left posterolateral thoracotomy.

Scenario 2

Stab wound to the right ventricle

0

A Correct answer

The right ventricle is the most anterior chamber of the heart and is thus most easily accessed via a median sternotomy.

Scenario 3

Bilateral penetrating chest injuries with lung parenchymal lacerations

0

C Correct answer

Bilateral penetrating lung injuries will often need wide exposure to be appropriately repaired. Access to both lungs is feasible via a median sternotomy, but a clamshell thoracotomy gives the best access to both hemithoraces.

THEME: Nerve damageA Horner’s syndromeB Neurapraxia of the common peroneal nerveC Neurapraxia of the median nerveD Neurapraxia of the radial nerveE Neurotmesis of the common peroneal nerveF Neurotmesis of the median nerveG Posterior interosseus nerve lesionH Sciatic nerve injury

For each of the patients listed below, select the site and type of nerve damage that best explains the clinical situation from the above list. Each option may be used once, more than once, or not at all.

The peroneal nerve is very susceptible to pressure, and thus may result in a neurapraxia. Following a supracondylar fracture of a child’s humerus, a neurapraxia of the median nerve is the most common neurological lesion but damage to the ulnar nerve is also not uncommon. If a surgical procedure is complicated by nerve injury, a transection (neurotmesis) must be considered – especially if the nerve has not been visualised. A flail upper limb suggests a brachial plexus lesion, which might well be associated with Horner’s syndrome if the sympathetic chain is involved. During a hip replacement, two forms of nerve injury are well documented: direct damage to the sciatic nerve at the level of the hip joint (more common); and pressure on the peroneal nerve at the neck of the fibula. During a posterior approach to the hip, the sciatic nerve is in particular danger.

Scenario 1

A 21-year-old man sustained a comminuted fracture of the right femur and a fracture of the ipsilateral tibia and fibula. He was treated with skeletal traction and a below-knee plaster overnight. On review it was noticed that he could not dorsiflex his right toes.

0

B Correct answer

Scenario 2

A child falls on an outstretched hand and sustains a severely displaced supracondylar fracture of the humerus.

0

C Correct answer

Scenario 3

Following a difficult elective plating of a non-union fracture of the humeral shaft, the patient was unable to extend his fingers and wrist. No nerves were visualised during the procedure.

0

D Correct answer

Scenario 4

A motorcyclist came off his bike at considerable speed. Both he and his bike were then dragged down the road by a car. On examination he had a flail left upper limb.

0

A Correct answer

Scenario 5

Following a total hip replacement performed via a posterior approach, the patient was noted to have a foot drop.

0

H Correct answer

THEME: bone conditions

A OsteoporosisB Scheuermann's kyphosisC Metastatic carcinomaD Osteoid osteomaE Giant cell tumour

For each of the patients described below, select the single most likely diagnosis from the options listed above. Each option may be used once, more than once or not at all.

 

Scenario 1

A 68-year-old frail lady suffered with back pain for 4 years and developed slight kyphosis in the last year. She underwent a hysterectomy and bilateral salpingo-ophorectomy at the age of 35. She presents with a fracture of the left neck femur after slipping at home.

0

A Correct answer

Osteoporosis can be caused by menopausal hormonal failure, Cushing’s disease, steroids, thyrotoxicosis and prolonged bedrest. Bone scan is diagnostic.

Scenario 2

A 30-year-old woman presents with pain and swelling in her thoracic region. Radiographs show a small lesion, with a well-demarcated central nidus surrounded by dense reactive bone in T6.

0

D Correct answer

An osteoid osteoma is a benign osteoblastic lesion. It typically occurs in 5–30-year-olds.

Scenario 3

An X-ray shows a large radiolucent zone in the head of the right humerus extending to the subchondral plate in a 29-year-old man, with localised pain.

0

E Correct answer

Giant cell tumours typically affect patients between 20–60 years of age. It occurs more commonly in women then men.

Scenario 4

A 70-year-old with a large tumour in left shoulder. He also complains of increasing difficulty swallowing and most recently breathing. On examination he has a large hard irregular thyroid swelling.

0

C Correct answer

Metastatic carcinoma is the commonest malignancy in bone. The majority originate from breast, prostate, lung, kidney and thyroid.

THEME: Aortic bypass grafting

A Aortobifemoral bypassB Axillobifemoral bypassC Femoral-to-femoral crossoverD Left iliac angioplastyE Left iliac angioplasty and femoral crossover

For each of the patients described below, select the procedure of choice from the list of options above. Each option may be used once, more than once, or not at all.

An aortobifemoral bypass graft has the highest patency rate of any bypass procedures to the femoral vessels. An axillobifemoral bypass graft should only be considered in the very high-risk surgical patient who has critical ischaemia. It should not be performed in patients with claudication.

Scenario 1

A 50-year-old man has a 50-yard (~45 m) claudication distance, with complete occlusion of the lower aorta, with patent femoral vessels.

0

A Correct answer

A – Aortobifemoral bypass

Scenario 2

A 79-year-old man with emphysema requires home oxygen. He has complete occlusion of the aorta, with patent femoral vessels. His toes appear gangrenous and dusky.

0

B Correct answer

B – Axillobifemoral bypass

Scenario 3

A 43-year-old postman, otherwise fit and well, who is a non-smoker presents with acute onset claudication in both feet – he is determined to go back to work. He has an aortic bifurcation block with good femoral run-off on both sides.

0

A Correct answer

A – Aortobifemoral bypass

THEME: SpleenA RuptureB Post-splenectomy C HypersplenismD CystE Splenosis

For each of the situations below, select the single most likely option from the list above. Each option may be used once, more than once or not at all.

 

Scenario 1

On examination, both flanks are dull to percussion. On the right flank, the dullness shifts when the patient is tilted to the left and, on the left flank, the dullness is fixed.

0

A Correct answer

The patient is demonstrating Ballance’s sign. This occurs when a large blood clot forms around a ruptured spleen with free intraperitoneal blood in the right flank. Left shoulder-tip pain can also occur from irritation of the left hemidiaphragm (Kehr’s sign).

Scenario 2

A full blood count (FBC) shows leucocytosis and thrombocytosis.

0

B Correct answer

Two to three days after splenectomy, there is a rise in the WCC and platelet count. The thrombocytosis may require prophylactic anticoagulation.

Scenario 3

An FBC shows anaemia, leucopenia and thrombocytopenia.

0

C Correct answer

The criteria for hypersplenism are (1) splenomegaly, (2) any combination of anaemia, leucopenia or thrombocytopenia, (3) bone marrow hyperplasia, and (4) improvement after splenectomy.

Scenario 4

There is recurrence of thrombocytopenia and Howell–Jolly bodies are not identified on the peripheral blood smear.

0

E Correct answer

Ectopic seeding of splenic tissue or accessory spleens may lead to recurrence after splenectomy. Red cells with cytoplasmic inclusions (Howell–Jolly bodies) are seen in the peripheral blood after splenectomy and its absence would indicate residual splenic tissue.

THEME: SHOCK

A Fat embolism

B Thromboembolism

C Cardiogenic shock

D Hypovolaemic shock

For each of the patients described below, select the single most likely diagnosis from the options listed above. Each option may be used once, more than once, or not at all.

 

Scenario 1

A 26-year-old man with a comminuted closed fracture of the femur shaft undergoes intra-medullary nail fixation. Two days after the operation, he develops a pyrexia, shortness of breath, petechial haemorrhages on his chest and tachycardia.

0

A Correct answer

Fat embolism occurs in patients with multiple closed fractures, but has been reported with other skeletal trauma, including burns. Fat embolism causes a temperature, tachycardia, shortness of breath, confusion and petechial haemorrhages, especially on the chest.

Scenario 2

A 72-year-old man with an underlying prostate carcinoma and a fractured femoral shaft undergoes intra-medullary nail fixation. Seven days after the operation, he develops shortness of breath, hypotension and tachycardia.

0

B Correct answer

Seven days post-operation is the characteristic time for a thromboembolism.

Scenario 3

A 60-year-old man develops sudden back pain. He is brought to Accident and Emergency (A&E) with a swollen tense abdomen, tachycardia, low volume pulse and low BP.

0

D Correct answer

In a patient with a sudden onset of back pain, a swollen tense abdomen and low BP, a ruptured aortic aneurysm is the most common diagnosis. This leads to hypovolaemic shock.

THEME: Types of ulcerationA Curling’s ulcer B Cushing’s ulcer C Marjolin’s ulcerD Neuropathic ulcersE Pyoderma gangrenosum

For each of the clinical scenarios listed below, select the lesion most likely to occur in that scenario from the above list. Each option may be used once, more than once, or not at all.

Marjolin’s ulcers are squamous cell carcinomas (SCCs) that develop in long-standing chronic venous ulcers. Gastroduodenal stress ulcers follow major trauma or sepsis. The pathophysiology is unclear, but is thought to relate to relative mucosal ischaemia, lack of oral alimentation and altered gastric mucous barrier function. Specific forms of gastroduodenal stress ulceration include: Cushing’s ulcer, which follows severe head injury; and Curling’s ulcer, which follows major burns. Pyoderma gangrenosum occurs in inflammatory bowel disease, and is more common in ulcerative colitis than Crohn’s disease.

Scenario 1

Squamous cell carcinoma in a chronic venous ulcer

0

C Correct answer

Scenario 2

Head injury

0

B Correct answer

Scenario 3

Major burns

0

A Correct answer

Scenario 4

Inflammatory bowel disease

0

E Correct answer

THEME: Suture material

A ‘0’Vicryl B 3‘0’ PDS C 6‘0’ PDS D 6‘0’ Prolene E 2‘0’ ProleneF 2‘0’ SilkG 1 PDSH Stainless steel wire

For each of the following situations, select the most appropriate suture material for the procedure from the above list. Each option may be used once, more than once, or not at all.

 

 

Scenario 1

Small bowel anastomosis.

0

B Correct answer

B – 3 ‘0’ PDS

Numerous suture materials can be used for bowel anastomoses. The most popular materials are the absorbable ones: PDS (polydioxanone) and Vicryl. Non-absorbable materials can also be used, including Ethibond. The size of the suture would vary from 2‘0’ to 4‘0’. Stapling techniques have become commonplace with the advantage of speed and a wider functional side-to-side anastomosis. There is no evidence that anastomotic leak rates are any higher between suturing and stapling.

Scenario 2

Mid-line abdominal wound closure.

0

G Correct answer

G – 1 PDS

The abdominal wall can be closed through a variety of techniques: layered closure or mass closure. The latter is quicker and currently the most popular technique in the UK. The suture material may be absorbable or non-absorbable, loop or non-loop. However, it must be strong and heavy (usually a 1 PDS or 1 Nylon).

Scenario 3

Sternotomy wound.

0

H Correct answer

H – Stainless steel wire

Sternotomy wounds in adults are closed with stainless steel wires. However, sternotomy wounds in paediatric patients are closed with a heavy PDS suture.

Scenario 4

Distal end of a below-knee femoropopliteal bypass.

0

D Correct answer

D – 6 ‘0’ Prolene

Arterial anastomoses are fashioned with prolene, an non-absorbable material, but gortex has also been used. For small calibre anastomoses fine material must be used, otherwise narrowing would occur.

Scenario 5

Securing a prosthetic mesh for incisional hernia repair.

0

E Correct answer

E – 2 ‘0’ Prolene

Prosthetic mesh techniques have become the accepted means for repairing incisional hernias. The aim is for the mesh to be secured in position and not migrate; therefore, a non-absorbable suture such as 2 ‘0’ prolene is most appropriate.

THEME: JaundiceA Hepatic jaundiceB Post-hepatic jaundiceC Pre-hepatic jaundice

For each of the clinical findings given below, select the correct type of jaundice from the above list. Each option may be used once, more than once, or not at all.

Pre-hepatic jaundice is associated with the presence of (urinary) urobilinogen and the absence of urinary bilirubin. Causes include recent blood transfusion and a family history of haemolytic syndromes. Hepatic jaundice may be conjugated or unconjugated, and is associated with a history of recent foreign travel, alcohol or drug abuse, ingestion of hepatotoxic drugs (halothane, chlorpromazine) or liver tumours. Post-hepatic jaundice is associated with bilirubin in the urine, a positive Courvoisier’s sign (extrahepatic duct obstruction), pruritus, a history of fever, jaundice and rigors (Charcot’s triad – due to ascending cholangitis) and a history of dark urine and pale stools.

Scenario 1

Bilirubin in the urine.

0

B Correct answer

B – Post-hepatic jaundice

Scenario 2

History of recent foreign travel.

0

A Correct answer

A – Hepatic jaundice

Scenario 3

Positive Courvoisier’s sign.

0

B Correct answer

B – Post-hepatic jaundice

Scenario 4

Associated pancreatitis.

0

B Correct answer

B – Post-hepatic jaundice

THEME: Thoracic trauma

A Pulmonary embolusB Cardiac tamponadeC Tension pneumothoraxD Haemothorax

For each of the following situations, select the most likely diagnosisfrom the above list. Each option may be used once, more than once, or not at all.

 

Scenario 1

A 29-year-old motorcyclist admitted following a road traffic accident. The patient sustained thoracic and pelvic injuries. A left intercostal drain has been inserted for pneumothorax. On transfer to the ward, the patient becomes acutely short of breath. He is hypotensive with engorged neck veins, the trachea is deviated to the right, with muffled heart sounds and reduced breath sounds on the left.

0

C Correct answer

This patient has a left tension pneumothorax despite the presence of a left intercostal drain. Chest drains may easily become blocked, kinked or clamped in error, with a resulting tension pneumothorax if there is an ongoing airleak.

Scenario 2

A 70-year-old man fell 15 ft from a ladder and landed on his back. The following

day on the ward he becomes short of breath, tachycardic and hypotensive. The

trachea is deviated to the right, breath sounds are reduced on the left and a loud

clicking prosthetic heart sound is heard.

0

D Correct answer

This patient has a mechanical prosthetic aortic valve (loud clicking second heart sound) and will thus be on warfarin. A fall has precipitated a left haemothorax, with reduced breath sounds on the left and a degree of mediastinal shift away from the affected side.

Scenario 3

A 35-year-old woman collapses on the ward 4 days after a thoracotomy and evacuation of haematoma following a stabbing to the right chest. She is tachypnoeic, tachycardic and severely hypotensive with engorged neck veins. The trachea is central, the heart sounds are normal. The post-operative intercostal drains are swinging with no air leak and minimal drainage.

0

A Correct answer

This patient becomes suddenly unwell 4 days after a major surgical procedure. The observations of the intercostal drains do not suggest a right pneumothorax . The other clinical features suggest acute right ventricular decompensation, most likely caused by a large pulmonary embolus. Tamponade is less likely with the site of injury and normal heart sounds.

THEME: LOWER-LIMB ULCERATION   A     Squamous cell carcinoma

B Hypertensive ulcer (Martorell syndrome) C Pyoderma gangrenosumD Neuropathic ulcerE Vasculitic ulcerF Necrobiosis lipoidicaG Sickle cell diseaseH Venous ulcerI Basal cell carcinomaJ Erythema nodosumFor each of the case descriptions below, select the most appropriate diagnosis from the list above.

 

Scenario 1

A 54-year-old woman with known inflammatory bowel disease presents with a large nodulopustular ulcerating lesion over her right anterior shin. It has a blue overhanging necrotic edge. Her ankle–brachial pressure indices are normal.

0

C Correct answer

Pyoderma gangrenosum (PG) are recurring nodulopustular ulcers commonly affecting the legs, abdomen and face. They are tender and have a red or blue overhanging necrotic edge. They heal with cribriform scars. Over 50% of patients with PG have associated underlying active or quiescent systemic disease – such as inflammatory bowel disease, seronegative rheumatoid arthritis, a lymphoproliferative disease, autoimmune hepatitis or Wegener’s granulomatosis. The diagnosis of PG is primarily clinical and by exclusion of other causes of cutaneous ulcerations with a similar appearance; skin histology may help. Treatment is usually with immunosuppressants and corticosteroids.

Scenario 2

A 57-year-old woman with varicose veins presents with a large ulcer over her left medial malleolus. This is associated with surrounding lipodermatosclerosis and eczema. The ankle–brachial pressure index in this leg is 1. The patient is moderately obese.

0

H Correct answer

Venous ulcers may develop spontaneously or following minor trauma. There may be a history of varicose veins, deep venous thrombosis, chronic venous insufficiency, poor calf-muscle function or arteriovenous fistulae. Other factors predisposing to venous ulceration include obesity, all risk factors for deep-vein thrombosis, a family history of varicose veins and a history of leg fracture. In long-term venous insufficiency, the skin undergoes changes (atrophy and variable pigmentation), with the dermis and subcutaneous tissue becoming indurated and fibrosed; this is termed lipodermatosclerosis. Venous eczema (erythema, scaling, weeping and itching) is a common feature associated with such ulceration; this is distinct from cellulitis. The ankle–brachial pressure indices are normal in patients with classic venous ulceration.

Scenario 3

A 59-year-old man presents with ulceration over the tips of his toes in his left foot and a large ulcer over his right heel. He also complains of paraesthesias in both feet. The ankle–brachial pressure indices are 1.05 on the right and 1.2 on the left. On neurological assessment, proprioception and vibration sense are reduced.

0

D Correct answer

Neuropathic ulcers can result from peripheral sensory neuropathy secondary to diabetes mellitus. The other causes of altered sensory neuropathy leading to neuropathic ulcers include spinal cord injuries, spina bifida, tabes dorsalis, syringomyelia, alcohol abuse and leprosy. Neuropathic ulcers are common at the sites of pressure or repeated trauma. The usual sites in the lower limb are the heads of the metatarsals, the interdigital clefts, the heel (calcaneum) and the lateral malleolus. In diabetic patients, the ankle–brachial pressure indices may be falsely elevated even in those with major vessel disease. This is due to calcification of the vessel walls and medial sclerosis. Neurological assessment may reveal altered sensation including proprioception and two-point discrimination in the foot or toes, reduced vibration sense and absent ankle jerks.

Scenario 4

A 30-year-old woman of Jamaican origin presents with a 4-month history of painful ulceration over her right shin. Ankle–brachial pressure indices are normal. The patient has mild splenomegaly. Her haemoglobin is 9.2 gm/dl.

0

G Correct answer

Sickle cell disease is an hereditary haemolytic anaemia occurring mainly among those of Afro-Caribbean origin. The haemoglobin S molecule crystallises during reduced blood oxygen tension, causing vascular occlusion. Depending on the affected vessel, patients may have bone or joint pain, priapism, neurological abnormalities or skin ulcers. The ulceration is due to occlusion of the cutaneous microcirculation, leading to ischaemia and skin breakdown. In dark-skinned individuals, there may be a non-specific increase in melanin pigmentation around the ulcer, which, if over the ankle region, may be confused with the discoloration caused by haemosiderin around a venous ulcer. Anaemia and splenomegaly are recognised features of sickle cell disease

THEME: Blood transfusion complicationsA Air embolism B Allergic reaction

C Bacterial contamination D Cannulation site thrombophlebitis E Circulatory overload F Delayed haemolytic transfusion reaction G Graft-versus-host disease H Hyperkalaemia I Hypocalcaemia J Hypothermia K Immediate haemolytic transfusion reactionL ImmunosuppressionM Iron overloadN Non-haemolytic febrile transfusion reactionO Post-transfusion purpuraP Transfusion-related acute lung injury (TRALI)Q Transfusion-transmitted infectionR Urticaria

The following patients all have complications of blood transfusion. Please select the most appropriate cause or description from the above list. The items may be used once, more than once, or not at all.

 

Scenario 1

You are called to see a 35-year-old multiparous woman on your surgical ward who, 6 days after an intra-operative blood transfusion, is complaining of nose bleeds and sacral bruising. A coagulation screen is normal. Her platelet count is 28 x 109/litre.

0

O Correct answer

O – Post-transfusion purpura (PTP)

PTP is defined as thrombocytopenia arising 5–12 days after transfusion of red cells. It is associated with the presence in the recipient of antibodies directed against the human platelet antigen (HPA) system. The patient becomes sensitised to foreign platelet antigen (commonly anti-PlA1), usually as a result of previous pregnancy or, more rarely, transfusion. If the patient then receives subsequent blood carrying this antigen, a secondary response occurs, leading to destruction of the recipient’s own platelets. Clinical signs are purpura and mucosal bleeding. Optimal treatment is by means of large doses of intravenous immunoglobulin, or plasma exchange.

Scenario 2

A 40-year-old man, with no history of pulmonary disease, develops acute dyspnoea within 16 h of receiving a whole blood transfusion. His oxygen saturations on air are 90%. Arterial blood gas pA(O2) is 9.0 kPa. A chest X-ray shows patchy shadowing in both lung fields.

0

P Correct answer

P – Transfusion-related acute lung injury (TRALI)

This man is suffering from TRALI. This is defined as acute dyspnoea with hypoxia and bilateral pulmonary infiltrates occurring within 24 h of transfusion. When there is no other apparent cause (too delayed to be air embolus) such symptoms point to this as the most likely diagnosis. TRALI is caused by agglutination of the patient’s white blood cells when they are exposed to potent HLA antibodies in the blood donor’s plasma. Diagnosis is supported by a positive cytotoxic cross-match where the donor’s plasma reacts to the patient’s white cells. Management consists of supportive measures, such as stopping the transfusion (if the reaction occurs while this is ongoing), resuscitation, oxygen therapy and Intensive Care Unit intervention, with or without ventilation as necessary.

Scenario 3

A 24-year-old female pedestrian is admitted unconscious to the Intensive Care Unit after a hit-and-run accident. She has been appropriately assessed and managed for her injuries. A blood transfusion has just been commenced. Having remained previously stable, you now notice that she is spiking a temperature of 39.5 °C, her blood pressure is 100/60 mmHg, and she is bleeding profusely through her wound dressings and oozing blood around her venepuncture sites. Her catheter contains dark urine that is dipstick positive for haemoglobinuria.

0

K Correct answer

K – Immediate haemolytic transfusion reaction

Clinical symptoms of a major transfusion reaction include urticaria, abdominal/lumbar/chest pain, flushing, headache, dyspnoea, vomiting and rigors. Clearly these signs are far easier to elicit in the conscious patient. In this scenario, however, the woman is not able to describe such symptoms. The signs in such a patient are therefore of utmost importance in diagnosing a haemolytic reaction. In the unconscious patient, hypotension (a fall > 10 mmHg) and generalised uncontrollable bleeding are the most important signs. Alongside these, further indications of an

immediate haemolytic reaction are fever (an increase of more than 1.5 °C above baseline), jaundice and haemoglobinuria. Immediate haemolytic transfusion reaction is usually the result of ABO incompatibility, leading to complement activation and massive intravascular haemolysis. Most severe reactions occur because of clerical error, hence all documentation should be re-checked and both patient and donor blood (from the bag in use) should undergo repeat cross-matching. The prime objective in the acute management of such a reaction is to maintain blood pressure and renal perfusion. This is achieved by immediate cessation of the transfusion and supportive resuscitation with intravenous fluids, titration of (1 : 10, 000 adrenaline as well as chlorpheniramine (10 mg) and hydrocortisone (100 mg). Renal failure and disseminated intravascular coagulation may lead to death in the worst case scenario.

Scenario 4

A 26-year-old Afro-Caribbean woman presents to her general practitioner (GP) with symptoms of lethargy and weight gain. She is known to have sickle cell anaemia and, because of multiple painful crises, has required regular blood transfusions. Blood tests requested by her GP show: thyroid-stimulating hormone 9 µmol/litre, free thyroxine 7 pmol/litre, calcium 2.15 mmol/litre. Further investigations reveal poor cardiac function.

0

M Correct answer

M – Iron overload

Repeated red cell transfusions (often given frequently as prophylaxis to sickle-cell sufferers having regular crises, to suppress haemoglobin S production) can lead to iron overload in the absence of chelation. Each unit of blood contains between 200 and 250 mg of iron. After 12 g of iron have been transfused (approximately 50 units of blood), clinical abnormalities start to appear. Unfortunately, organ damage will have occurred earlier than this. Iron damages the liver, the endocrine organs (leading to failure of growth, delayed or absent puberty, diabetes mellitus, hypothyroidism and hypoparathyroidism) and the myocardium. Parenteral treatment with desferrioxamine can prevent iron overload, if compliance with medication is good. In the absence of intensive iron chelation, death often occurs in the second or third decade, usually from congestive heart failure or cardiac arrhythmias.

THEME: Life-threatening chest injuries A Traumatic aortic ruptureB Massive haemothorax (right)C Tension pneumothorax (left)D Cardiac tamponade 

For each of the physical signs described below, select the most likely diagnosis from the list of conditions above. Each option may be used once, more than once or not at all.

Tension pneumothorax and cardiac tamponade produce similar signs in that they cause distension of the neck veins (on inspiration = Kussmaul’s sign), cyanosis and hypotension. Tension pneumothorax and traumatic aortic rupture are usually due to blunt trauma. A tension pneumothorax would deviate the trachea away from the side of the injury as would a traumatic aortic rupture. Traumatic aortic rupture classically produces a widened mediastinum with obliteration of the aortic knob and the presence of pleural capping.

Scenario 1

Beck’s triad

0

D Correct answer

Cardiac tamponade

Scenario 2

Tracheal deviation to the right

0

C Correct answer

Tension pneumothorax (left)

Scenario 3

Most commonly caused by penetrating right chest injury

0

B Correct answer

Massive haemothorax (right)

THEME: Consent for surgical treatment

A Apply to make the child a ward of courtB No, surgery cannot proceedC Obtain consent from patientD Obtain consent from wifeE Yes, surgery can proceed

For each of the following situations, select the most likely answer from the above list. Each option may be used once, more than once, or not at all.

Scenario 1

A 24-year-old man found unconscious by the roadside is brought to hospital by ambulance. It is evident that the patient’s condition is rapidly deteriorating because of an expanding extradural haematoma and he is unable to give consent. His wife had been contacted and is at the hospital but has expressed her refusal to allow him to be operated on. Would you proceed against her wishes?

0

E Correct answer

E – Yes, surgery can proceed

Surgery is needed for the preservation of life and can be performed despite the patient’s inability to give consent. The patient’s wife cannot give permission or stop her husband’s operation. No adult can act as legal proxy for any other in the UK with regard to giving consent for surgical treatment.

Scenario 2

A member of an extreme religious sect has brought their 11-year-old son to hospital with generalised peritonitis from a perforated appendix. The child’s condition deteriorates and he needs a laparotomy which both parents adamantly refuse, saying that he will recover through the fervent prayers of members of the sect. Despite repeated attempts by the surgical team to persuade the parents of this child, they refuse to give consent for the surgical treatment that the surgeon deems to be essential. What option is available to the surgeon?

0

A Correct answer

A– Apply to make the child a ward of court

The surgeon can either respect the patient’s wishes and not subject the child to an operation or can make an application for the child to be made a ward of court and proceed with appropriate surgical treatment. In this case surgical treatment is essential and almost all surgeons would do the latter.

Scenario 3

A 70-year-old woman with severe psychiatric illness is undergoing compulsory psychiatric treatment, having been sectioned under the Mental Health Act. She has a fall in the psychiatry unit and sustains a fracture of the neck of the right femur for which she is referred forsurgical treatment The orthopaedic surgeon thinks that internal fixation of the fracture is the best management, in agreement with her psychiatrist. Her psychiatric state does not allow her to give informed consent for surgery. What process should follow?

0

E Correct answer

E – Yes, surgery can proceed

The Mental Health Act does not allow for the compulsory treatment of any medical condition other than a mental disorder. The orthopaedic surgeon may, however, proceed with surgery for the patient’s fractured femur if he or she and the patient’s psychiatrist agree that it is the best form of management for her. It is good clinical practice to also obtain a second consultant’s surgical opinion, confirming the need for operative treatment and to involve the relatives in the decision-making process where possible. Every clinician should make detailed entries in the patient’s records and sign and date them.

Scenario 4

A 63-year-old man with a brain tumour refuses any surgery and expresses his wish formally in writing. He is judged to be mentally competent. The following day he enters into a coma and his wife, who was abroad, arrives at his bedside and demands that surgical treatment is commenced. Can surgery proceed?

0

B Correct answer

B – No, surgery cannot proceed

No. Before the deterioration of this patient’s condition he clearly refused to consent to operative treatment. Therefore, surgery cannot be performed even when he is unable to express his refusal for such a seemingly essential intervention.

THEME: Sterilisation and disinfection A AlcoholB AutoclaveC Boiling waterD Chlorhexidine-cetrimide (Savlon)E Chlorhexidine in detergent (Hibiscrub)F Dry heatG Ethylene oxideH Gamma radiationI GlutaraldehydeJ IodineK Steam with formaldehyde

The above are all methods of either sterilisation or disinfection. For each of the following please select the most appropriate answer from the above list. The items may be used once, more than once, or not at all.

 

Scenario 1

A method of sterilising sutures.

0

G Correct answer

G – Ethylene oxide

Sterilisation is described as the complete destruction of all viable microorganisms. Heat-sensitive equipment, sutures and other single-use items are prepared using ethylene dioxide as part of an industrial process. The gas is toxic but effective at killing vegetative spores, bacteria and viruses.

Scenario 2

A method used for disinfecting endoscopes.

0

I Correct answer

I – Glutaraldehyde

Joseph Lister pioneered the concept of antisepsis in 1867 while he was Professor of Surgery at the Glasgow Royal Infirmary. He was able to reduce post-operative infections with the use of carbolic acid spray. Modern day antiseptic techniques have now moved on but the essential principles are the same. Disinfection is defined as a reduction in the number of viable organisms and is synonymous with antisepsis except that disinfectants are used in non-living tissue. Glutaraldehyde is rapidly active against bacteria and viruses including hepatitis B virus and human immunodeficiency virus but is less effective against spores and Mycobacterium tuberculosis. However, steam with formaldehyde (another more toxic aldehyde) can be used as a method of sterilisation.

Scenario 3

A method of sterilisation that is monitored using the Bowie–Dick test.

0

B Correct answer

B – Autoclave

Autoclaving is a sterilisation technique that uses saturated steam at high pressure and kills all organisms including heat-resistant spores, Mycobacterium tuberculosis and viruses. The holding time describes the minimum amount of time at a set temperature that guarantees sterility. The Bowie–Dick test is the colour change seen on a strip of heat-sensitive tape (diagonally-striped) attached to wrapped instruments that monitors steam penetration associated with the sterilisation process. Each use of the autoclave is documented in a printout that is, however, the only absolute method of guaranteeing sterility.

THEME: Neonatal surgical diagnoses

A Duodenal atresia B Exomphalos C Gastroschisis D Hirschsprung’s disease E Imperforate anusF Mid-gut volvulusG Necrotising enterocolitis (NEC)H Oesophageal atresiaI Tracheo-oesophageal fistula

For each of the clinical scenarios given below, select the most likely diagnosis from the above list. Each option may be used once, more than once, or not at all.

 

Scenario 1

A male infant with trisomy 21, born at term, presents with abdominal distension, bile-stained vomiting and collapse. There has been no passage of meconium.

0

D Correct answer

D – Hirschsprung’s disease

Hirschsprung’s disease usually presents in the neonatal period with a history of delayed passage of meconium, abdominal distension and bilestained vomiting. The infant may be shocked with impending enterocolitis. Hirschsprung’s disease is more common in children with Down’s syndrome (trisomy 21). It can be differentiated postnatally from duodenal atresia in which there is passage of meconium, even in complete atresia.

Scenario 2

An antenatal observation of intestine and liver outside the fetal abdomen.

0

B Correct answer

B – Exomphalos

Both gastroschisis and exomphalos are frequently diagnosed antenatally. In exomphalos, the liver is frequently outside the abdomen and the herniated organs are covered by an identifiable membrane.

Scenario 3

A previously well male infant, 6 weeks old and born at 35 weeks gestation, presents with bile-stained vomiting.

0

F Correct answer

F – Mid-gut volvulus

Bile-stained vomiting in a previously well infant should raise the suspicion of a mid-gut volvulus.

Scenario 4

A 4-hour-old female infant noted to be ‘frothy’, was born to a mother with polyhydramnios during pregnancy.

0

H Correct answer

H – Oesophageal atresia

A baby with oesophageal atresia is frequently born to a mother with polyhydramnios. At birth the infant is unable to swallow even saliva, which froths through the mouth and nose.

Scenario 5

An 8-day-old female infant presents with respiratory distress at each oral feed.

0

I Correct answer

I – Tracheo-oesophageal fistula

Oesophageal atresia can occur with or without a tracheo-oesophageal fistula, which itself can occur in isolation. When an infant feeds, there may be aspiration of milk through the fistula into the lungs. Occasionally, the infant may be several months old before the diagnosis is made.

THEME: Cervical lymphadenopathy A Idiopathic histiocytic necrotising lymphadenitis

B Infectious mononucleosisC Lymphoreticular diseaseD Metastatic malignancyE SarcoidosisF Scalp infectionG TonsillitisH ToxoplasmosisI Tuberculosis 

The following scenarios describe various presentations of cervical lymphadenopathy. From the above list choose the most likely cause. Each item may be chosen once, more than once, or not at all.

The causes of cervical lymphadenopathy can be classified into four groups:

primary malignancy (lymphomas and leukaemias, ie lymphoreticular disease)

metastatic malignancy (local carcinomas) infections sarcoid

Scenario 1

A 68-year-old man presents with an asymptomatic slowly growing painless lump in the neck. On examination he has a hard 2-cm mass lying laterally in the submandibular triangle of his neck, deep to the middle third of the right sternocleidomastoid muscle. The patient is noted to have a dysphonia.

0

D Correct answer

D – Metastatic malignancy

This position describes that of the cervical lymph nodes that drain the oropharynx and larynx. The lymphatic system of the neck can be divided into Levels 1–5. This man has palpable Level 3 nodes that also drain Levels 1 and 2, which receive lymph from the scalp, face and lips. The most likely diagnosis here is a laryngeal carcinoma given the dysphonia.

Scenario 2

A 12-year-old girl presents with recurrent sore throats and on examination she has a soft 2-cm mass lying laterally within the anterior triangle of the neck, just below the angle of the mandible.

0

G Correct answer

G – Tonsillitis

This acute history and anatomical description of involved lymph nodes is typical of tonsillitis, remembering that the tonsil itself is a lymphoid structure. Other acute causes of cervical lymphadenopathy include pharyngitis/laryngitis and Epstein–Barr virus infection. Chronic causes include tuberculosis and sarcoidosis.

Scenario 3

A 28-year-old Caucasian presents with a 3-month history of night sweats, weight loss and a unilateral enlargement of his left tonsil. There is also a rubbery enlarged level III lymph node in the left posterior triangle.

0

C Correct answer

C – Lymphoreticular disease

This is fairly obvious given the history. The differential would be tuberculosis/human immunodeficiency virus in at-risk individuals.

THEME: Disorders of calcium homeostasis

A Acute pancreatitisB Addison’s diseaseC Chronic renal failureD Ectopic parathyroid hormoneE Familial hypocalciuric hypercalcaemiaF HyperparathyroidismG HypoparathyroidismH MalignancyI Milk–alkali syndromeJ Multiple myelomaK OsteomalaciaL PseudohypoparathyroidismM SarcoidosisN ThiazidesO Thyrotoxicosis

P Vitamin D deficiency

The following scenarios describe patients with disorders of calcium metabolism. Choose from the above list the most likely cause. Each item may be chosen once, more than once, or not at all.

When assessing calcium values it is important to establish albumin levels because only 40% of total calcium is ionised and therefore physiologically relevant. The remainder is bound to albumin and so unavailable to tissues. Routine laboratory values usually measure total plasma calcium and this must be corrected for the serum albumin concentration. This is done by adding or subtracting 0.02 mmol/litre for every g/litre by which the albumin lies below or above 40 g/litre. For critical values, the sample should be taken from an uncuffed arm with the patient in the fasting state to reduce variability in the protein values.

Scenario 1

A 45-year-old woman is 24 h post-parathyroidectomy and biopsy of her remaining gland. She complains of cramps in her hands and around her mouth. Her plasma calcium is 1.95 mmol/litre and serum albumin is 42 g/litre.

0

G Correct answer

G – Hypoparathyroidism

This woman is truly hypocalcaemic, given her normal albumin levels, and needs urgent treatment. Her hypocalcaemia may be the result of removal of the dominant gland (so leaving under-active tissue) of oedema, damage to the remaining glands, or indeed of inadvertent surgical removal of all the parathyroid glands. She requires immediate intravenous 10 ml calcium gluconate given slowly over 5 min with cardiac monitoring. This infusion then needs to be repeated with 40 ml over 24 h. Maintenance therapy is achieved with 1a-(OH)D3 or 1,25(OH)D3. The most common general cause of hypocalcaemia, however, is renal failure. Signs include Chvostek’s and Trousseau’s: the former being twitching of the facial muscles induced by tapping over the facial nerve, and the latter being a carpopedal spasm caused by occluding the brachial artery.

Scenario 2

A 55-year-old woman is seen in the urology clinic after attending Casualty with recurrent stones. She comments that she is also undergoing treatment for peptic ulceration. Her history is unremarkable, and she is otherwise well and takes no other regular medication. Her plasma calcium is found to be 3.2 mmol/litre with an albumin of 47 g/litre.

0

F Correct answer

F – Hyperparathyroidism

This woman demonstrates two of the symptoms of hypercalcaemia, which are renal calculi or calcification, bone pain, muscle weakness, intestinal atony, psychosis, peptic ulceration and pancreatitis (‘stones, groans and psychic moans’). The list of causes of hypercalcaemia is extensive, however, in an otherwise well patient, the two most likely diagnoses are primary hyperparathyroidism or malignancy. The latter appears unlikely, and a diagnosis of hyperparathyroidism can be made on estimation of parathyroid hormone levels. Hyperparathyroidism is described as being primary, secondary, or tertiary. Primary is usually caused by a single parathyroid adenoma, rarely a carcinoma. Secondary is seen as a physiological response to hypocalcaemia, eg renal failure or vitamin D deficiency (calcium is low or normal). Tertiary develops in response to secondary hyperparathyroidism and is caused by autonomous parathyroid hyperplasia.

Scenario 3

A 65-year-old man presents with severe back pain and recurrent bleeding from his gums. He also mentions that his vision has become increasingly blurred in recent weeks. His plasma calcium is 2.95 mmol/litre.

0

J Correct answer

J – Multiple myeloma

This man’s hypercalcaemia is related to the excessive osteoclast activity caused by the neoplastic clonal proliferation of marrow plasma cells. These abnormal plasma cells produce abnormal immunoglobulins (paraproteins) that may be associated with the excretion of light chains in the urine, which are known as Bence Jones proteins. H. Bence Jones (1814–73) studied medicine at St Georges Hospital, London, and was not only an excellent physician but also a skilled chemist. His insistence of examination of urine both chemically and microscopically was truly ahead of its time. Monoclonal (M) bands on serum protein electrophoresis provide evidence of such proteins and form the cornerstone of diagnosing this condition. Patients present in their 60s with bone pain, anaemia, recurrent infections and bleeding. Renal failure ensues, caused by

deposition of light chains in the tubules, hypercalcaemia, hyperuricaemia and amyloid deposition. Paraproteins increase blood viscosity causing blurred vision and gangrene.

THEME: Disorders of bone

A Achondroplasia B Craniocleidodysostosis C Diaphyseal aclasis D Ollier’s disease E Osteochondrodystrophy F Osteogenesis imperfectaG OsteopetrosisH Perthe’s diseaseI RicketsJ Scurvy

For each of the following situations, select the most appropriate cause for the presentation from the above list. Each option may be used once, more than once, or not at all.

 

Scenario 1

A 13-year-old boy is brought to the orthopaedic outpatient clinic with a history of tiredness, recurrent throat and chest infections and gradual loss of hearing. X-ray reveals a ‘marble bone’ appearance.

0

G Correct answer

G – Osteopetrosis

Osteopetrosis is an autosomal recessive condition. The patient, usually a young adult, may present with symptoms of anaemia (tiredness) or thrombocytopenia (recurrent throat and chest infections) because of decreased marrow space. Deafness and optic atrophy can result from compression of the cranial nerves. Blood investigations may reveal a leukoerythroblastic picture. The bones are very dense and brittle, and X-ray reveals a lack of differentiation between the cortex and the medulla described as ‘marble bone’.

Scenario 2

A 14-year-old boy, who is small for age, is brought to his GP with loss of hearing in both ears. On examination, he has a blue sclera, knock-knees, and hypermobile fingers. X-rays show multiple fractures (old) of the long bones and irregular patches of ossification.

0

F Correct answer

F – Ostegenesis imperfecta

Osteogenesis imperfecta (brittle bone disease) is defective osteoid formation due to the congenital inability to produce adequate intercellular substances, such as osteoid, collagen and dentine. In addition, there is a failure of maturation of collagen in all the connective tissues. Some typical clinical features of this condition include: a broad skull, blue sclera, premature deafness, scoliosis, ligament laxity, coxa vara and knock knees. X-rays may reveal translucent bones, multiple fractures particularly of the long bones, wormian bones (irregular patches of ossification), and a trefoil pelvis.

Scenario 3

A 3-year old boy is brought to the GP surgery with a swollen and painful right knee joint. The parents also say that they recently noticed some bleeding from his gums. He lies still and refuses to move the limb. X-ray shows generalised rarification of the bones in his legs.

0

J Correct answer

J – Scurvy

Scurvy (vitamin C deficiency) causes a failure of collagen synthesis and osteoid formation. The patient, usually a child or an infant, may present with swelling and tenderness near the large joints. There may be bleeding from the gums as they are spongy. Spontaneous bleeding may lead to subperiosteal haematoma and the child remains still (pseudoparalysis) as a result of subperiosteal bleeding. X-rays shows generalised bone rarefaction. The metaphysis may be deformed or fractured. Vitamin C in the form of ascorbic acid should be given in doses of 1 gm/day and the child should be encouraged to eat fresh fruit and vegetables.

Scenario 4

A 2-year-old infant is brought to the Emergency Department with convulsions. On examination, the child lies listless and flaccid, appears small for age and there is

noticeable thickening of both wrists. X-ray shows an increase in the depth and width of the epiphysis of the lower ends of the radius and ulna.

0

I Correct answer

I – Rickets

Rickets is the childhood form of osteomalacia. Because of vitamin D deficiency osteoid fails to ossify. Symptoms start from about the age of 1 year. The child may present with tetany or convulsions. The child is small for age and there is a history of failure to thrive. Bony deformities include: bowing of the femur and tibia, deformity of the skull (craniotabes), deformity of the chest wall with thickening of the costochondral junction (ricketty rosary), and a transverse sulcus in the chest caused by the pull of the diaphragm (Harrison’s sulcus). The characteristic X-ray change is an increase in the depth and width of the epiphysis and the adjacent metaphysis has a ‘cupped’ appearance; these changes are most noticeable in the wrist.

THEME: Investigation of urinary tract disorders A Antegrade ureteropyelography B Computed tomography (CT)C Computed tomography urography (CTU)D Cystourethroscopy E DMSA (dimercaptosuccinic acid) scanF Intravenous ureterogram (IVU) G Kidney, ureter and bladder (KUB) X-ray H MAG 3 scan I  Magnetic resonance imaging (MRI)J Renal angiographyK Retrograde ureteropyelographyL Transrectal prostatic biopsyM Transrectal ultrasound (TRUS)N Ultrasound kidneys, ureters and bladderO Urethrography and cystographyP Urodynamic studies 

The above are all examples of investigations used in the diagnosis of urinary tract disorders. Please pick the most appropriate investigation for the following clinical presentations/descriptions. The items may be used once, more than once, or not at all.

 

Scenario 1

A 62-year-old man presents to The Emergency Department complaining of haematuria, gnawing right loin pain and malaise. Abdominal examination does not reveal anything grossly abnormal. Urine cytology and culture are normal. An ultrasound is performed urgently and shows a complex right renal cyst with a calcified wall and a more solid central component. The patient tells you that he has had a prior allergic reaction to an injected contrast that he had in his 50s for some form of abdominal investigation. How should he be further investigated?

0

I Correct answer

I – Magnetic resonance imaging (MRI)

The standard primary investigation for haematuria and loin pain (with or without a mass) is an ultrasound, which has already been undertaken for this patient. If a renal cyst demonstrates a solid intracystic element or an irregular or calcified wall, as in this case, it is regarded as potentially malignant and requires further imaging for staging. This is usually completed by computed tomography (CT) scanning of the chest and abdomen, with intravenous contrast administration. In patients who have a prior history of reaction to contrast, MRI is the chosen modality for staging assessment. In the staging of patients with renal carcinoma MRI appears more accurate in delineating inferior vena cava or renal vein involvement, compared with CT.

Scenario 2

A 40-year-old woman who is an inpatient on the Urology Ward recently had a left-sided percutaneous nephrostomy tube sited for pyonephrosis. This was performed as an emergency procedure on her admission with sepsis and right loin pain, following an ultrasound KUB showing left-sided hydronephrosis. Urea 15 mmol/litre; creatinine 205 µmol/litre. What further imaging should be undertaken to assess the cause of her infected kidney?

0

C Correct answer

C – Computed tomography urography (CTU)

Although this woman has a nephrostomy tube sited and this could be used for antegrade contrast studies of her urinary tract, she has raised urea and creatinine that could be exacerbated by contrast use. CTU has the advantage in this scenario of allowing rapid visualisation of the renal tract without the need for contrast injection. It has a high sensitivity for ureteric stones. If no renal tract stone is identified, CTU has the advantage over intravenous urogram, and retrograde or antegrade

ureteropyelography, in that it can demonstrate other intra-abdominal pathologies that might be causing upper tract obstruction.

Scenario 3

A 50-year-old smoker presents to the urology outpatients after referral by his general practitioner for painless haematuria. His prostate feels smooth and of normal size and he denies any lower urinary tract symptoms. All relevant blood tests and urine samples have been taken and show nothing of significance other than confirming haematuria. You need to arrange the next investigation.

0

F Correct answer

F – Intravenous ureterogram (IVU)

The gold standard first investigation (cystourethroscopy may often be required subsequently) for investigating painless macroscopic haematuria is still IVU. Although many urology departments may combine plain abdominal kidney, ureter and bladder X-ray with renal tract ultrasound it is important to recognise that ultrasonography is less sensitive than IVU for detecting tumours of the upper tract, which comprise 1–2% of all urothelial tumours.

THEME: Vitamin deficiency disorders

A 1,25-DihydroxycholecalciferolB -TocopherolC RiboflavinD Vitamin B12E NiacinF MenaquinoneG RetinolH ThiaminI Ascorbic acid

For each of the options above, choose the single most likely vitamin deficiency disorder from the list below. Each option may be used once, more than once, or not at all.

 

Scenario 1

Angular stomatitis

0

C Correct answer

Scenario 2

Subacute combined degeneration of the cord

0

D Correct answer

Scenario 3

Xerophthalmia

0

G Correct answer

Scenario 4

Osteomalacia

0

A Correct answer

Scenario 5

Dementia

0

H Correct answer

Scenario 6

Pellagra

0

E Correct answer

Scenario 7

Scurvy

0

I Correct answer

Scenario 8

Beri beri

0

H Correct answer

Scenario 9

Seborrhoeic dermatitis

0

C Correct answer

Scenario 10

Coagulation defects

0

F Correct answer

THEME: Management of breast cancer

A AnastrozoleB Axillary clearanceC ChemotherapyD Excision of marginsE MastectomyF RadiotherapyG TamoxifenH Wide local excisionI Wide local excision and axillary clearanceJ Wire-guided wide local excision

The following descriptions are of patients with breast cancer. Please select the most appropriate treatment option. The items may be used once, more than once, or not at all.

 

Scenario 1

A 57-year-old has been seen for follow-up with a history of a 2-cm lump in the upper outer quadrant of the right breast. Mammography and ultrasound are highly suspicious for breast cancer (R5, U5) and this is confirmed on fine-needle aspiration cytology (FNAC; C5) and core biopsy, which demonstrates invasive ductal carcinoma. Subsequent investigations reveal no evidence of metastatic disease.

0

I Correct answer

I – Wide local excision and axillary clearance

Breast conservation surgery is considered the optimal therapy for local treatment of breast cancer and commonly takes the form of a wide local excision for local treatment of breast cancer. When combined with postoperative radiotherapy the results are comparable to mastectomy.

Indications for wide local excision include:

stage I or II disease single primary lesion tumours < 4 cm in diameter (this is because poor cosmetic results often follow

wide local excision for tumours > 4cm; however tumours over 4 cm may be treated by wide local excision if the breast is large).

Evidence of vascular invasion requires not only local treatment, but also treatment directed towards the axillary lymph nodes. This is most commonly performed by

means of an axillary clearance. Following surgery, the patient should be referred for consideration of chemotherapy. Note: the practice of sentinel node biopsy is becoming a popular alternative and is now in fact recommended in every woman with breast cancer (RCS guidelines). Using a dye technique, if the sentinel node is found to be cancer-free women can be spared axillary surgery.

Scenario 2

A 45-year-old woman is seen following mastectomy and axillary clearance. She has made an uneventful recovery. The final histology report states ‘invasive ductal carcinoma, completely excised with two out of 17 lymph nodes demonstrating infiltration with tumour. Oestrogen and progesterone receptor negative’. She asks you what further treatment is required.

0

C Correct answer

C – Chemotherapy

Combination chemotherapy reduces the annual odds of recurrence and death. The absolute benefits are greatest for women with node-positive disease. Furthermore, chemotherapy should be discussed with and offered to woman with node-negative but grade III (poorly differentiated) tumours. It is uncertain as to which regimen is best and there are several ongoing trials. At present the commonest combination is cyclophosphamide, methotrexate and 5-fluorouracil (CMF).

Scenario 3

A 53-year-old woman is seen in the clinic for results of biopsies from a screen-detected lesion in her right breast. Cytology from an FNA is graded as C4 and core biopsies have demonstrated the presence of intermediate grade ductal carcinoma in situ (DCIS).

0

J Correct answer

J – Wire-guided wide local excision

Treatment of screen-detected lesions differs from palpable breast cancer in that radiological localisation by means of a wire inserted into the lesion is required before excision. This patient has ductal carcinoma in situ (DCIS), the majority of which are now detected in screening programmes. In patients with extensive DCIS (> 4 cm) or disease affecting more than one quadrant a mastectomy should be performed. Surgical

staging of the axilla is not required. Following adequate local excision of DCIS, patients should be considered for radiotherapy to the breast, which may reduce the risk of recurrence and development of invasive breast cancer.

Scenario 4

An 89-year-old lady has been brought to the one-stop clinic by her daughter who noted a lump in her mother’s left breast while dressing her. Her mother is wheelchair-bound, suffers with dementia and is normally a resident of a nursing home. On examination the patient is a frail woman with a large mass in the upper outer quadrant of her left breast. The mass is fixed to the skin and underlying muscle and, in addition, she has fixed lymph nodes in her left axilla. Core biopsies demonstrate an invasive ductal carcinoma, which is oestrogen and progesterone receptor positive.

0

G Correct answer

G – Tamoxifen

The situation described is not unusual and management of locally advanced (stage III) disease involves treatment with systemic therapy. This may take the form of chemotherapy or hormonal treatment. Surgery and/or radiotherapy should normally be given afterwards, the timing of which is dependent on response to therapy. In patients who are unfit for surgery and who are unsuitable candidates for chemotherapy (as in the case described), the aim of therapy is to control the primary tumour while maintaining the best quality of life. In this instance, primary treatment is with tamoxifen provided the receptor status of the tumour is known. Anastrozole (Arimidex) is used as a second-line treatment.

Scenario 5

A 45-year-old woman has undergone a wide local excision and axillary clearance for invasive ductal carcinoma. The final histology reads ‘invasive ductal carcinoma with 2-mm clearance from the nearest (lateral margin). There is no evidence of lymph node metastases. Oestrogen and progesterone receptor negative’.

0

F Correct answer

F – Radiotherapy

Treatment with radiotherapy is mandatory in the case of a young woman who has undergone breast conservation surgery. This is usually started 2–4 weeks post-

operatively, provided that wounds have healed and an adequate local excision has been performed (clear lateral margins of > 1 mm). The extent of the margin needs to be balanced against the final cosmetic appearance; however, any margin that is incomplete, ie < 1 mm, requires further excision before radiotherapy.

THEME: Management of jaundiceA Endoscopic retrograde cholangiopancreatography (ERCP) and endoscopic stentingB CystenterostomyC CholecystectomyD Roux-en-Y hepaticojejunostomyE Pancreaticoduodenectomy F Palliative treatment

For each of the situations below, select the single most likely option from the above list. Each option may be used once, more than once or not at all.

 

Scenario 1

A 30-year-old woman presents with right upper quadrant pain and jaundice. Investigation with abdominal ultrasonography and ERCP shows marked cystic dilatation of the extrahepatic bile duct, with distal irregularity and stenosis but without intrahepatic dilatation.

0

D Correct answer

A choledochal cyst is a congenital segmental cystic dilatation of the biliary tree. In type I cysts (dilatation of the extrahepatic bile duct), treatment involves complete excision of the cyst and restoration of the biliary drainage using a roux-en-Y hepaticojejunostomy. Choledochal cysts are premalignant and biliary carcinoma can arise from incomplete resection.

Scenario 2

A 68-year-old man presents with jaundice, anorexia and recent weight loss, but with no pruritus or fever. Abdominal ultrasonography shows ductal dilatation and no gallstones. An ERCP shows stenosis involving both the pancreatic and the biliary duct (‘double-duct’ sign). CT demonstrates a 3 cm mass within the head of the pancreas with no evidence of invasion or spread.

0

E Correct answer

Preoperative biliary drainage in patients undergoing pancreaticoduodenectomy should be selective (eg in patients with cholangitis or predicted delay in surgery) because it has been associated with higher rates of complications. The ‘double-duct’ sign is highly indicative of pancreatic cancer but can also be present in chronic pancreatitis.

Scenario 3

A 65-year-old man presents with painless jaundice and episodic right upper quadrant pain. Abdominal ultrasonography and CT show dilatation of the extrahepatic bile duct. Endoscopic ultrasonography and ERCP show a mass at the ampulla confined to the mucosa. Biopsy reveals an adenocarcinoma.

0

E Correct answer

Ampullary tumours present relatively early with jaundice and are most commonly adenocarcinomas. Pancreaticoduodenectomy is the preferred treatment for patients with potentially curable disease. Other treatments include transduodenal excision or endoscopic stenting for patients who either are unable to have a pancreaticoduodenectomy or have more advanced disease.

Scenario 4

A 62-year-old woman presents with right upper quadrant pain, jaundice and nausea. Ultrasonography shows a stone in Hartmann’s pouch, causing narrowing of the common hepatic duct. Similarly, ERCP shows extrinsic compression of the common hepatic duct with intrahepatic duct dilatation, no fistula and no common bile duct stones.

0

C Correct answer

Mirizzi’s syndrome occurs when impaction of a gallstone in Hartmann’s pouch or the cystic duct results in extrinsic compression of the common hepatic duct and obstructive jaundice. Progressive erosion of the surrounding wall leads to the formation of a fistula into the bile duct. Where no fistula is present, a total cholecystectomy or a subtotal cholecystectomy with closure of the gallbladder neck remnant may be performed.

THEME: Mean graft survival rates at one year following transplantation

A > 90%B 90%C 80%D 70%E 60%F 50%G 40%

For each of the clinical scenarios listed below, select the closest survival rate. Each option may be used once, more than once, or not at all.

 

Scenario 1

A 20-year-old man had a living donor renal allograft transplant.

0

A Correct answer

Living donor grafts have a slightly better survival rate than cadaveric grafts.

Scenario 2

A 30-year-old woman has an uncomplicated liver transplant for primary biliary cirrhosis.

0

C Correct answer

This can vary with patient factors and indications for transplant.

Scenario 3

A 22-year-old man undergoes a lung transplant for cystic fibrosis.

0

D Correct answer

The International Society for Heart and Lung Transplantation report 1-year survival rates of 71%, and 5-year survival rates of 45% following lung transplantation.

Scenario 4

A 40-year-old woman with type 1 diabetes mellitus has a pancreas transplant.

0

E Correct answer

Pancreatic graft survival continues to improve. Graft survival is highest when the pancreas is transplanted in combination with a renal allograft.

THEME: Antibiotics A Co-amoxiclavB SulphonamidesC AminoglycosidesD CephalosporinsE PolymyxinsF TetracyclinesG Co-trimoxazoleH Rifampicin 

For each of the statements below select the most likely antibiotic from the list above. Each option may be used once, more than once or not at all.

 

Scenario 1

Contraindicated in pregnant women

0

F Correct answer

Tetracyclines

Scenario 2

Act as inhibitors of nucleic acid synthesis in bacteria

0

H Correct answer

Rifampicin

Scenario 3

Effective against both aerobic and anaerobic bacteria

0

A Correct answer

Co-amoxiclav

Scenario 4

Used with caution and close monitoring of levels in patients with renal impairment

0

C Correct answer

Aminoglycosides

Scenario 5

Used for prophylaxis in abdominal surgery

0

D Correct answer

Cephalosporins

Scenario 6

Effective against chlamydial pelvic infections

0

F Correct answer

Tetracyclines

Scenario 7

No longer advisable for urinary tract infections in the elderly

0

G Correct answer

Co-trimoxazole